Re: aiming to complete Everett's derivation of the Born Rule

158 views
Skip to first unread message

Brent Meeker

unread,
Apr 15, 2022, 2:41:03 PM4/15/22
to Everything List


On 4/14/2022 2:00 PM, George Kahrimanis wrote:
On Wednesday, April 13, 2022 at 8:55:48 PM UTC+3 meeke...@gmail.com (Brent) wrote:

Decoherence has gone part way in solving the when/where/what basis questions, but only part way.

As I wrote at the end of my first reply to your message, I share your concern about decoherence but I see the glass as half-full; that is, with a little more subtlety I hope that the matter can be formulated in clear terms.

Surely collapse is easier to handle as a general concept (except, on the other hand, that it requires new dynamics). I forgot to mention that my argument for deriving the Born Rule works with collapse, too -- so it is an alternative to Gleason's theorem.

Here I define colapse as an irreversible process, violating unitarity of course, and I keep it separate from randomisation. The latter means that each outcome is somehow randomised -- an assumption we can do without.

Collapse can also be described in a many-world formulation! It differs from the no-collapse MWI only in being irreversible.

If you can throw away low probability branches, what's to stop you from throwing away all but one?  You've already broken unitary evolution.  If you read Hardy's axiomatization of QM you see that the difference between QM and classical mechanics turns on a single word in Axiom 5 Continuity: There exists a continuous reversible transformation on a system between any two pure states of that system.

My argument in outline is
1. assessment that MWI-with-collapse is workable;
2. therefore, outcomes of small enough measure can be neglected in practice;

Yes, I've wondered if a smallest non-zero probability could be defined consistent with the data.

3. now Everett's argument can proceed, concluding that the Born Rule is a practically safe assumption (to put it briefly).

So I have replaced two assumptions of Gleason's theorem, randomisation and non-contextuality, by the assessment of workability only.

If you don't feel comfortable yet with formulating collapse in a many-world setting, let us also assume randomisation (God plays dice), for the sake of the argument, in a single-world formulation. That is, we ASSUME the existence of probability; then the previous argument just guarantees that this probability follows the Born Rule.

Assume?  Randomness is well motivated by evidence.  And it's more random than just not knowing some inherent variable, because in the EPR experiment a randomized hidden variable can on explain the QM result if it's non-local.


Of course I favour the first version of the argument, using the many-world formulation of collapse, to avoid the "God plays dice" nightmare.

Why this fear of true randomness?  We have all kinds of classical randomness we just attributed to "historical accident".  Would it really make any difference it were due to inherent quantum randomness?  Albrect and Phillips have made an argument that there is quantum randomness even nominally classical dynamics. https://arxiv.org/abs/1212.0953v3

Brent


Thanks for the comments so far, because they stirred my thinking and motivated fresh ideas, some of which I hope will prove helpful and worth discussing, if and when they mature.

George K.
--
You received this message because you are subscribed to the Google Groups "Everything List" group.
To unsubscribe from this group and stop receiving emails from it, send an email to everything-li...@googlegroups.com.
To view this discussion on the web visit https://groups.google.com/d/msgid/everything-list/06930c0c-5537-4fb7-bf70-fd8c7d9859b0n%40googlegroups.com.

Alan Grayson

unread,
Apr 16, 2022, 11:34:57 AM4/16/22
to Everything List
On Friday, April 15, 2022 at 12:41:03 PM UTC-6 meeke...@gmail.com wrote:


On 4/14/2022 2:00 PM, George Kahrimanis wrote:
On Wednesday, April 13, 2022 at 8:55:48 PM UTC+3 meeke...@gmail.com (Brent) wrote:

Decoherence has gone part way in solving the when/where/what basis questions, but only part way.

As I wrote at the end of my first reply to your message, I share your concern about decoherence but I see the glass as half-full; that is, with a little more subtlety I hope that the matter can be formulated in clear terms.

Surely collapse is easier to handle as a general concept (except, on the other hand, that it requires new dynamics). I forgot to mention that my argument for deriving the Born Rule works with collapse, too -- so it is an alternative to Gleason's theorem.

Here I define colapse as an irreversible process, violating unitarity of course, and I keep it separate from randomisation. The latter means that each outcome is somehow randomised -- an assumption we can do without.

Collapse can also be described in a many-world formulation! It differs from the no-collapse MWI only in being irreversible.

If you can throw away low probability branches, what's to stop you from throwing away all but one?  You've already broken unitary evolution.  If you read Hardy's axiomatization of QM you see that the difference between QM and classical mechanics turns on a single word in Axiom 5 Continuity: There exists a continuous reversible transformation on a system between any two pure states of that system.

My argument in outline is
1. assessment that MWI-with-collapse is workable;
2. therefore, outcomes of small enough measure can be neglected in practice;

Yes, I've wondered if a smallest non-zero probability could be defined consistent with the data.

3. now Everett's argument can proceed, concluding that the Born Rule is a practically safe assumption (to put it briefly).

So I have replaced two assumptions of Gleason's theorem, randomisation and non-contextuality, by the assessment of workability only.

If you don't feel comfortable yet with formulating collapse in a many-world setting, let us also assume randomisation (God plays dice), for the sake of the argument, in a single-world formulation. That is, we ASSUME the existence of probability; then the previous argument just guarantees that this probability follows the Born Rule.

Assume?  Randomness is well motivated by evidence.  And it's more random than just not knowing some inherent variable, because in the EPR experiment a randomized hidden variable can on explain the QM result if it's non-local.


Of course I favour the first version of the argument, using the many-world formulation of collapse, to avoid the "God plays dice" nightmare.

Why this fear of true randomness?  We have all kinds of classical randomness we just attributed to "historical accident".  Would it really make any difference it were due to inherent quantum randomness?  Albrect and Phillips have made an argument that there is quantum randomness even nominally classical dynamics. https://arxiv.org/abs/1212.0953v3

True randomness implies unintelligibility; that is, no existing physical process for causing the results of measurements. AG 

Brent Meeker

unread,
Apr 16, 2022, 3:44:09 PM4/16/22
to everyth...@googlegroups.com


On 4/16/2022 8:34 AM, Alan Grayson wrote:
Of course I favour the first version of the argument, using the many-world formulation of collapse, to avoid the "God plays dice" nightmare.

Why this fear of true randomness?  We have all kinds of classical randomness we just attributed to "historical accident".  Would it really make any difference it were due to inherent quantum randomness?  Albrect and Phillips have made an argument that there is quantum randomness even nominally classical dynamics. https://arxiv.org/abs/1212.0953v3

True randomness implies unintelligibility; that is, no existing physical process for causing the results of measurements. AG 

"It happened at random in accordance with a Poisson process with rate parameter 0.123" seems perfectly intelligible to me.  There is a physical description of the system with allows you to predict that, including the value of the rate parameter.  It only differs from deterministic physics in that it doesn't say when the event happens.

I always wonder if people who have this dogmatic rejection of randomness understand that quantum randomness is very narrow.  Planck's constant is very small and it introduces randomness, but with a definite distribution and on certain variables.  It's not "anything can happen" as it seems some people fear.

Brent

Alan Grayson

unread,
Apr 16, 2022, 5:58:56 PM4/16/22
to Everything List
Every single trial is unintelligible. AG

Brent Meeker

unread,
Apr 16, 2022, 7:03:55 PM4/16/22
to everyth...@googlegroups.com
I find that remark unintelligble.  I don't think "intelligble" means what you think it means.

Brent

Alan Grayson

unread,
Apr 16, 2022, 7:24:57 PM4/16/22
to Everything List
It means there exists no definable physical process to account for the outcome of a single trial. AG 

Brent Meeker

unread,
Apr 16, 2022, 7:46:09 PM4/16/22
to everyth...@googlegroups.com
That's what is usually called "non-deterministic".  "Unintelligble" means not understandable or incomprehensible. 

Brent


Alan Grayson

unread,
Apr 16, 2022, 8:08:53 PM4/16/22
to Everything List
I think you're fooling yourself if you think a non-determinsitic process is comprehensible. AG

Brent Meeker

unread,
Apr 16, 2022, 8:34:51 PM4/16/22
to everyth...@googlegroups.com
Consider the converse.  When you comprehend some physical evolution, is it essential that it be deterministic.  Every event has many causes, do you have to know every one of them to comprehend it?  Think of all the things you would have to say did NOT happen in order that your comprehension be complete.  The way I look at it, we call classical mechanics deterministic only because most of the time there are a few (not a bazillion) factors we can approximately determine in advance, so that an almost certain prediction, within a range of uncertainty, is possible.  Even within strict determinism there are at this very moment gamma rays from distant supernova approaching you and which cannot be predicted but which might influence your thoughts and instruments.

Brent
--
You received this message because you are subscribed to the Google Groups "Everything List" group.
To unsubscribe from this group and stop receiving emails from it, send an email to everything-li...@googlegroups.com.

Alan Grayson

unread,
Apr 17, 2022, 10:11:45 AM4/17/22
to Everything List
A simple example of your point is a gas at some temperature and pressure, confined in some volume. For a given particle in the ensemble, we can't determine its exact path because we lack information about its interactions. But if we had that knowledge, we could determine its exact path, and any uncertainties in that information would translate into uncertainties in its path. But inherent randomness in QM is different and probably has nothing to do with the UP. For example, for a small uncertainty in position, there is a large uncertainty in velocity, so we can get simultaneous measurements of position and velocity, but the latter will manifest large fluctuations for succeeding measurements. Thus, the "inherent randomness" in QM is the assumption that every individual trial or outcome of a measurement is UNcaused; that is, the particular outcome can't be traced to some prior state -- what AE called God playing dice with the universe. AG
 
On Saturday, April 16, 2022 at 6:34:51 PM UTC-6 meeke...@gmail.com wrote:; 

Brent Meeker

unread,
Apr 17, 2022, 6:01:03 PM4/17/22
to everyth...@googlegroups.com


On 4/17/2022 7:11 AM, Alan Grayson wrote:
A simple example of your point is a gas at some temperature and pressure, confined in some volume. For a given particle in the ensemble, we can't determine its exact path because we lack information about its interactions. But if we had that knowledge, we could determine its exact path, and any uncertainties in that information would translate into uncertainties in its path. But inherent randomness in QM is different and probably has nothing to do with the UP.
Did you read the paper I cited?:  https://arxiv.org/abs/1212.0953v3

Brent

Alan Grayson

unread,
Apr 17, 2022, 7:11:58 PM4/17/22
to Everything List
No. I didn't read your original post on this thread. But I see the authors assume quantum fluctuations, and therefore deny causalty. You get what you pay for. In my example, there surely are caused probabilities, even if we don't have complete understanding of the initial conditions. But why address my issue if a link satisfies you? AG

Alan Grayson

unread,
Apr 17, 2022, 7:15:19 PM4/17/22
to Everything List
I meant, of course, CAUSALITY. AG

Brent Meeker

unread,
Apr 17, 2022, 8:19:44 PM4/17/22
to everyth...@googlegroups.com
The authors point out that the Heisenberg uncertainty principle limits the accuracy of determining initial conditions even if the physics of evolution is perfectly deterministic.

I addressed your issue because you posted it here...as a courtesy.  If you don't want it addressed...why post it.

Brent

Alan Grayson

unread,
Apr 17, 2022, 9:33:04 PM4/17/22
to Everything List
I was aware of the limitation on precision implied by the HUP. I was addressing whether simultaneous measurements are possible despite the HUP. I think they are possible. But my main point is that acausality is tantamount to unintelligible. IMO, there's a huge difference between being unable to perfectly predict the time evolution of a system, and it being uncaused. AG

Brent Meeker

unread,
Apr 17, 2022, 11:16:34 PM4/17/22
to everyth...@googlegroups.com


On 4/17/2022 6:33 PM, Alan Grayson wrote:
I was aware of the limitation on precision implied by the HUP. I was addressing whether simultaneous measurements are possible despite the HUP. I think they are possible.

The HUP directly refers ideal measurements which are preparations.  Each destructive measurement can simultaneously measure conjugate variables to arbitrary precision.  But repeating the destructive measurements on exactly the same prepared system will then give a scatter of answers which satisfies the HUP.


But my main point is that acausality is tantamount to unintelligible. IMO, there's a huge difference between being unable to perfectly predict the time evolution of a system, and it being uncaused. AG

Is there?  Even if the unpredicitability is in-principle?  What is the huge difference?

Brent

spudb...@aol.com

unread,
Apr 18, 2022, 7:34:46 AM4/18/22
to meeke...@gmail.com, everyth...@googlegroups.com
Not to choke the flow of the convo, but some years ago Albrecht also worked on the Observer issue and time via re-examining the work of Ludwig Boltzmann and his Boltzmann Brain. Please continue.
Message has been deleted

Alan Grayson

unread,
Apr 18, 2022, 8:35:22 AM4/18/22
to Everything List
On Sunday, April 17, 2022 at 9:16:34 PM UTC-6 meeke...@gmail.com wrote:


On 4/17/2022 6:33 PM, Alan Grayson wrote:
I was aware of the limitation on precision implied by the HUP. I was addressing whether simultaneous measurements are possible despite the HUP. I think they are possible.

The HUP directly refers ideal measurements which are preparations.  Each destructive measurement can simultaneously measure conjugate variables to arbitrary precision.  But repeating the destructive measurements on exactly the same prepared system will then give a scatter of answers which satisfies the HUP.


But my main point is that acausality is tantamount to unintelligible. IMO, there's a huge difference between being unable to perfectly predict the time evolution of a system, and it being uncaused. AG

Is there?  Even if the unpredicitability is in-principle?  What is the huge difference?

Brent

 So what, in your view, bugged AE about probability in QM? AG

Brent Meeker

unread,
Apr 18, 2022, 2:06:04 PM4/18/22
to everyth...@googlegroups.com


On 4/18/2022 5:35 AM, Alan Grayson wrote:
But my main point is that acausality is tantamount to unintelligible. IMO, there's a huge difference between being unable to perfectly predict the time evolution of a system, and it being uncaused. AG

Is there?  Even if the unpredicitability is in-principle?  What is the huge difference?

Brent

 So what, in your view, bugged AE about probability in QM? AG

I asked you first.

Brent

Alan Grayson

unread,
Apr 18, 2022, 2:17:45 PM4/18/22
to Everything List
IIRC, you asked what was bugging ME, not AE. My guess is that he thought acausality violated locality and/or realism. For example, the Pilot Wave theory assumes each particle has a definite position and momentum. It doesn't violate the HUP because the HUP simply limits what we can measure. AG

Alan Grayson

unread,
Apr 18, 2022, 2:29:47 PM4/18/22
to Everything List
Or maybe AE objected to the Copenhagen view that properties don't exist prior to measurement. This would certainly be acausal since all processes take finite time intervals to occur, which Copenhagen implicitly denies. AG 

Brent Meeker

unread,
Apr 18, 2022, 2:32:36 PM4/18/22
to everyth...@googlegroups.com
I asked you  "What is the huge difference?"  Which you ignored and just asked another question.

Brent

sce...@libero.it

unread,
Apr 18, 2022, 2:45:39 PM4/18/22
to everyth...@googlegroups.com

A deterministic clockwork universe vs a lawless universe (see Svozil, Arxiv,2000). I think QM is in between.

--
Inviato da Libero Mail per Android

Lunedì, 18 Aprile 2022, 02:35PM +02:00 da Alan Grayson agrays...@gmail.com:

Alan Grayson

unread,
Apr 18, 2022, 3:55:56 PM4/18/22
to Everything List
But the difference is obvious and implied. Whereas the resultant probabilties attained might be indistinguishable, the underlying realities are clearly distinct, say between Copenhagen and deBroglie-Bohm (Pilot Wave theory). Since, at heart, you're an instrumentalist, I assume the distinction for you is meaningless.  AG 

Alan Grayson

unread,
Apr 18, 2022, 4:18:32 PM4/18/22
to Everything List
And you ignored the additional information/interpretation that I offered. Oh, FWIW, according to Scerir QM is half-pregnant. AG 

Brent Meeker

unread,
Apr 18, 2022, 5:18:43 PM4/18/22
to everyth...@googlegroups.com
You can invent arbitrarily many theories of "distinct underlying realities" which are empirically indistinguishable...that's why they are just interpretations.  The only use I see for interpretations with no empirical difference is they may suggest better theories.  I see no other reason to prefer one interpretation over another.  You might as well introduce fairies into an interpretation or ask Deepak Chopra which one is really real.

Brent

George Kahrimanis

unread,
Apr 18, 2022, 6:45:25 PM4/18/22
to Everything List
On Monday, April 18, 2022 at 3:35:22 PM UTC+3 agrays...@gmail.com wrote:

So what, in your view, bugged AE about probability in QM? AG

I think I have come to a crisp understanding of this issue, which I want to submit to you. However, we must take into consideration that the notion of probability many scientists have these days is very different from the one implied in Einstein's comment "God doesn't play dice".

Einstein seems to have a good old-fashioned understanding of probability based on rolling the dice, shuffling the deck, and so on, which has also been formalised as "Kolmogorov complexity". That is, a shuffling complicated enough to make it technically impossible to run the needed calculations in the next 15 seconds, say, in which I am obliged to play my hand. Of course I trust that no other players in this game can run such calculations in the prescribed time (I trust with "moral certainty", not with absolute certainty).

This outlook of probability is incompatible with certain currently popular views of probability. For one, entropy considerations are irrelevant in general, unless when they just describe shuffling in other words. So-called Bayesian priors are also baseless strictly speaking, though they do serve in a "let us try this" approach.

One more notion to shed is that of propability issuing from ANY theoretical probabilistic model, for example conventional QM. (Surely, if you are comfortable with the latter, then Einstein's comment is meaningless!) I cite an important (I think) philosophical work by Wolfgang Schwarz: "No Interpretation of Probability" Erkenntnis 83, 1195–1212 (2018), <https://doi.org/10.1007/s10670-017-9936-9>. He argued that such models do NOT issue probability; they issue just numbers which the users ACCEPT AS probabilities -- in whatever interpretation of probability one assumes as fundamental. This is the key to understanding Einstein's comment.

So, in plain words, Einstein's comment means the following. If the interpretation of QM treats normalised measures as probabilities, we need to understand this in terms of our basic notion of probability, that is shuffling the deck or rolling the dice. So in each measurement someone must roll dice or something, in order that probability will arise. Since QM does not allow for such a mechanism, we are left to trusting that probabilities issued by QM are as good AS IF generated by a randomising mechanism (of a familiar kind). This "as if" creates a doubt whether the notion of probability from QM is equivalent to that from shuffling. This is not a silly question, because it has relevance to decision theory (in particular, on whether Maximisation of Expected Utility is a rationally justified method).

George K.

Alan Grayson

unread,
Apr 18, 2022, 8:08:48 PM4/18/22
to Everything List
So if someone, like Bohr, comes up with a lawless universe, that's fine with you; or do you deny the lawlessness? AG

Brent Meeker

unread,
Apr 18, 2022, 9:27:31 PM4/18/22
to everyth...@googlegroups.com
A lawless universe that is predictable per the Schroedinger equation??  What does that mean?

Brent

John Clark

unread,
Apr 19, 2022, 6:43:00 AM4/19/22
to 'Brent Meeker' via Everything List
On Mon, Apr 18, 2022 at 2:17 PM Alan Grayson <agrays...@gmail.com> wrote:

> the Pilot Wave theory assumes each particle has a definite position and momentum.

That's true but unlike Many Worlds Pilot Wave theory is non-local, it postulates there is a mysterious force of some sort that is undiminished by distance in which two particles billions of light years apart can INSTANTLY affect each other without affecting anything in between. It seems to me if that were the case then we'd have to know everything before we could know anything, and that does not conform with observation because although we don't know everything we do know some things. If the universe was really non-local we couldn't even make approximate predictions regardless of if things were deterministic or not.

Copenhagen assumes a particle has NO position and momentum if it has not been measured. Pilot Wave theory assumes  a particle has ONE position and momentum if it has not been measured. Many Worlds assumes Schrodinger's equation means what it says so a particle has EVERY position and momentum the equation allows regardless of if it has been measured or not.

> It doesn't violate the HUP because the HUP simply limits what we can measure.

Then you should like Many Worlds because it says everything happens because of Schrodinger's equation, and Schrodinger's equation is 100% deterministic. Many Worlds also explains why that, although from the multiverse point of view things are as deterministic as Schrödinger's equation, to any particular observer in one of those worlds there would be a limit to how accurate his predictions can be.

John K Clark    See what's on my new list at  Extropolis
tpw


Alan Grayson

unread,
Apr 19, 2022, 11:31:47 AM4/19/22
to Everything List
On Tuesday, April 19, 2022 at 4:43:00 AM UTC-6 johnk...@gmail.com wrote:
On Mon, Apr 18, 2022 at 2:17 PM Alan Grayson <agrays...@gmail.com> wrote:

> the Pilot Wave theory assumes each particle has a definite position and momentum.

That's true but unlike Many Worlds Pilot Wave theory is non-local, it postulates there is a mysterious force of some sort that is undiminished by distance in which two particles billions of light years apart can INSTANTLY affect each other without affecting anything in between. It seems to me if that were the case then we'd have to know everything before we could know anything, and that does not conform with observation because although we don't know everything we do know some things. If the universe was really non-local we couldn't even make approximate predictions regardless of if things were deterministic or not.

Copenhagen assumes a particle has NO position and momentum if it has not been measured. Pilot Wave theory assumes  a particle has ONE position and momentum if it has not been measured. Many Worlds assumes Schrodinger's equation means what it says so a particle has EVERY position and momentum the equation allows regardless of if it has been measured or not.

> It doesn't violate the HUP because the HUP simply limits what we can measure.

Then you should like Many Worlds because it says everything happens because of Schrodinger's equation, and Schrodinger's equation is 100% deterministic.

We've discussed this before, many times. If S's equation represented a horse race, with probabilities changing during the race -- of the order of final results -- why do you think the race continues in other worlds, with all combinations of outcomes? I think you egregiously misinterpret what S's equation is telling us.  AG

Many Worlds explains why that, although from the multiverse point of view things are as deterministic as Schrödinger's equation, to any particular observer in one of those worlds there would be a limit to how accurate his predictions can be.

John Clark

unread,
Apr 19, 2022, 11:53:06 AM4/19/22
to 'Brent Meeker' via Everything List
On Tue, Apr 19, 2022 at 11:31 AM Alan Grayson <agrays...@gmail.com> wrote:

>  If S's equation represented a horse race, with probabilities changing during the race -- of the order of final results -- why do you think the race continues in other worlds, with all combinations of outcomes?

Because until Alan Grayson sees the end of the race, or somebody tells Alan Grayson about it, Alan Grayson can't be certain what world Alan Grayson is in. Alan Grayson could be in a world where horse X won or Alan Grayson could be in a world where horse Y won, until Alan Grayson receives more information Alan Grayson would have to say the odds are 50-50.

Yes I know the above sounds awkward but, as I've said many times before, when discussing matters of this sort personal pronouns have no uniquely specified subject.

> We've discussed this before, many times.

We certainly have.  
John K Clark    See what's on my new list at  Extropolis
yui





 


--
You received this message because you are subscribed to the Google Groups "Everything List" group.
To unsubscribe from this group and stop receiving emails from it, send an email to everything-li...@googlegroups.com.

Alan Grayson

unread,
Apr 19, 2022, 12:08:15 PM4/19/22
to Everything List
On Tuesday, April 19, 2022 at 9:53:06 AM UTC-6 johnk...@gmail.com wrote:
On Tue, Apr 19, 2022 at 11:31 AM Alan Grayson <agrays...@gmail.com> wrote:

>  If S's equation represented a horse race, with probabilities changing during the race -- of the order of final results -- why do you think the race continues in other worlds, with all combinations of outcomes?

Because until Alan Grayson sees the end of the race, or somebody tells Alan Grayson about it, Alan Grayson can't be certain what world Alan Grayson is in. Alan Grayson could be in a world where horse X won or Alan Grayson could be in a world where horse Y won, until Alan Grayson receives more information Alan Grayson would have to say the odds are 50-50.

Not necessarily 50-50. Depends on how many horses are in the race. But much more important is you claim, without argument, that all possible outcomes are realized, most or many in other worlds. Never any justification of that MIS-interpretation of probability. AG 

John Clark

unread,
Apr 19, 2022, 12:24:51 PM4/19/22
to 'Brent Meeker' via Everything List
On Tue, Apr 19, 2022 at 12:08 PM Alan Grayson <agrays...@gmail.com> wrote:

> you claim, without argument, that all possible outcomes are realized.

I don't need to make an argument for that because the one and only assumption that Many Worlds makes, perhaps "axiom" would be a better word, is that Schrödinger's Equation means what it says. If you want to claim that all outcomes allowed by Schrödinger are NOT realized then you would have to introduce another axiom (a.k.a. assumption) that restricts that number. But I remind you that Arkham's razor says the simplest theory that explains observations is the one to be preferred.

 
John K Clark    See what's on my new list at  Extropolis
ems


Alan Grayson

unread,
Apr 19, 2022, 12:42:24 PM4/19/22
to Everything List
On Tuesday, April 19, 2022 at 10:24:51 AM UTC-6 johnk...@gmail.com wrote:
On Tue, Apr 19, 2022 at 12:08 PM Alan Grayson <agrays...@gmail.com> wrote:

> you claim, without argument, that all possible outcomes are realized.

I don't need to make an argument for that because the one and only assumption that Many Worlds makes, perhaps "axiom" would be a better word, is that Schrödinger's Equation means what it says.

But S's equation just gives the time dependent probabilities BEFORE a measurement is taken. You've added an additional postulate without any justification.  AG

John Clark

unread,
Apr 19, 2022, 12:59:20 PM4/19/22
to 'Brent Meeker' via Everything List


On Tue, Apr 19, 2022 at 12:42 PM Alan Grayson <agrays...@gmail.com> wrote:

>> I don't need to make an argument for that because the one and only assumption that Many Worlds makes, perhaps "axiom" would be a better word, is that Schrödinger's Equation means what it says.

> But S's equation just gives the time dependent probabilities BEFORE a measurement is taken. You've added an additional postulate without any justification.  AG

Schrödinger's Equation is time independent, it works just as well forwards or backwards, so "before" or "after" are irrelevant terms. And Schrödinger makes no use of "measurement" and says nothing about it, although it's easy enough to logically extrapolate from Schrödinger's axiom and conclude that "measurement" simply means self location. If you insist measurement does something more than that you're going to have to add another axiom explaining exactly what that "something" is and exactly what it does.
 
John K Clark    See what's on my new list at  Extropolis
m4x



--
You received this message because you are subscribed to the Google Groups "Everything List" group.
To unsubscribe from this group and stop receiving emails from it, send an email to everything-li...@googlegroups.com.

Brent Meeker

unread,
Apr 19, 2022, 2:05:35 PM4/19/22
to everyth...@googlegroups.com
That's not quite right though.  According the SE there are no outcomes.  The world vector in Hilbert space just keeps rotating around in the infinite dimensional space and by some means there are preferred bases such that in those bases sort-of-classical thing seem to happen...but don't really because it's all reversible.

Brent

 
John K Clark    See what's on my new list at  Extropolis
ems


--
You received this message because you are subscribed to the Google Groups "Everything List" group.
To unsubscribe from this group and stop receiving emails from it, send an email to everything-li...@googlegroups.com.

Alan Grayson

unread,
Apr 19, 2022, 3:01:15 PM4/19/22
to Everything List
On Tuesday, April 19, 2022 at 10:59:20 AM UTC-6 johnk...@gmail.com wrote:


On Tue, Apr 19, 2022 at 12:42 PM Alan Grayson <agrays...@gmail.com> wrote:

>> I don't need to make an argument for that because the one and only assumption that Many Worlds makes, perhaps "axiom" would be a better word, is that Schrödinger's Equation means what it says.

> But S's equation just gives the time dependent probabilities BEFORE a measurement is taken. You've added an additional postulate without any justification.  AG

Schrödinger's Equation is time independent,

Then why, for example, does the solution for a free particle spread out as time progresses? AG 

John Clark

unread,
Apr 19, 2022, 4:43:47 PM4/19/22
to 'Brent Meeker' via Everything List
On Tue, Apr 19, 2022 at 3:01 PM Alan Grayson <agrays...@gmail.com> wrote:

>> Schrödinger's Equation is time independent,

> Then why, for example, does the solution for a free particle spread out as time progresses? AG 

As time progresses things change, that is in fact what time means. So if something spreads out as time progresses if you reverse time then that "something" would converge. Schrodinger's wave equation works in either direction, no information is lost so if you know what the wave looks like now you can figure out what it will look like tomorrow and also figure out what it looked like yesterday.   

John K Clark    See what's on my new list at  Extropolis
ptp

Alan Grayson

unread,
Apr 19, 2022, 5:46:51 PM4/19/22
to Everything List
Do us all a big favor and stop the BS'ing. Solutions to the SE wouldn't be time-dependent unless the SE is time-dependent. It also has a time-independent form, which IIRC, is when it can be solved by separation of variables. AG 

John Clark

unread,
Apr 19, 2022, 5:55:20 PM4/19/22
to 'Brent Meeker' via Everything List
On Tue, Apr 19, 2022 at 5:46 PM Alan Grayson <agrays...@gmail.com> wrote:
 
> the SE wouldn't be time-dependent unless the SE is time-dependent.

I can't argue with that, and a banana wouldn't be a banana unless a banana was a banana
 
John K Clark    See what's on my new list at  Extropolis
beb
p

smitra

unread,
Apr 19, 2022, 7:16:30 PM4/19/22
to everyth...@googlegroups.com
An issue here is that the different QM interpretations are actually
different theories that do make different predictions for certain
experiments that one can at least in principle perform. Bohm theory
invokes quantum equilibrium, without which the Born rule will be
violated. And CI and MWI make different predictions for Deutsch-type
experiments.

While it's conventional to refer to these as interpretations, it's
similar to calling special relativity and ether theory different
interpretations of electromagnetism when effects of order (v/c)^2 would
be too small to detect.

Saibal



> Brent
>
> --
> You received this message because you are subscribed to the Google
> Groups "Everything List" group.
> To unsubscribe from this group and stop receiving emails from it, send
> an email to everything-li...@googlegroups.com.
> To view this discussion on the web visit
> https://groups.google.com/d/msgid/everything-list/c0c67139-4239-ad0d-d7be-dc0f2783e4c1%40gmail.com
> [1].
>
>
> Links:
> ------
> [1]
> https://groups.google.com/d/msgid/everything-list/c0c67139-4239-ad0d-d7be-dc0f2783e4c1%40gmail.com?utm_medium=email&utm_source=footer

Alan Grayson

unread,
Apr 19, 2022, 7:45:06 PM4/19/22
to Everything List
On Tuesday, April 19, 2022 at 3:55:20 PM UTC-6 johnk...@gmail.com wrote:
On Tue, Apr 19, 2022 at 5:46 PM Alan Grayson <agrays...@gmail.com> wrote:
 
> the SE wouldn't be time-dependent unless the SE is time-dependent.

I can't argue with that, and a banana wouldn't be a banana unless a banana was a banana

And it's also not worth arguing with a shameless liar who distorts my comment. A'hole; go back and read it again. AG 

smitra

unread,
Apr 19, 2022, 11:45:27 PM4/19/22
to everyth...@googlegroups.com
On 18-04-2022 23:18, Brent Meeker wrote:

Alan Grayson

unread,
Apr 20, 2022, 7:21:47 PM4/20/22
to Everything List


On Friday, April 15, 2022 at 12:41:03 PM UTC-6 meeke...@gmail.com wrote:


On 4/14/2022 2:00 PM, George Kahrimanis wrote:
On Wednesday, April 13, 2022 at 8:55:48 PM UTC+3 meeke...@gmail.com (Brent) wrote:

Decoherence has gone part way in solving the when/where/what basis questions, but only part way.

As I wrote at the end of my first reply to your message, I share your concern about decoherence but I see the glass as half-full; that is, with a little more subtlety I hope that the matter can be formulated in clear terms.

Surely collapse is easier to handle as a general concept (except, on the other hand, that it requires new dynamics). I forgot to mention that my argument for deriving the Born Rule works with collapse, too -- so it is an alternative to Gleason's theorem.

Here I define colapse as an irreversible process, violating unitarity of course, and I keep it separate from randomisation. The latter means that each outcome is somehow randomised -- an assumption we can do without.

Collapse can also be described in a many-world formulation! It differs from the no-collapse MWI only in being irreversible.

If you can throw away low probability branches, what's to stop you from throwing away all but one?  You've already broken unitary evolution.  If you read Hardy's axiomatization of QM you see that the difference between QM and classical mechanics turns on a single word in Axiom 5 Continuity: There exists a continuous reversible transformation on a system between any two pure states of that system.

My argument in outline is
1. assessment that MWI-with-collapse is workable;
2. therefore, outcomes of small enough measure can be neglected in practice;

Yes, I've wondered if a smallest non-zero probability could be defined consistent with the data.

3. now Everett's argument can proceed, concluding that the Born Rule is a practically safe assumption (to put it briefly).

So I have replaced two assumptions of Gleason's theorem, randomisation and non-contextuality, by the assessment of workability only.

If you don't feel comfortable yet with formulating collapse in a many-world setting, let us also assume randomisation (God plays dice), for the sake of the argument, in a single-world formulation. That is, we ASSUME the existence of probability; then the previous argument just guarantees that this probability follows the Born Rule.

Assume?  Randomness is well motivated by evidence.  And it's more random than just not knowing some inherent variable, because in the EPR experiment a randomized hidden variable can on explain the QM result if it's non-local.



Of course I favour the first version of the argument, using the many-world formulation of collapse, to avoid the "God plays dice" nightmare.

Why this fear of true randomness?  We have all kinds of classical randomness we just attributed to "historical accident".  Would it really make any difference it were due to inherent quantum randomness?  Albrect and Phillips have made an argument that there is quantum randomness even nominally classical dynamics. https://arxiv.org/abs/1212.0953v3

The authors regard quantum fluctuations as fundamental. How are they defined? AG

Brent


Thanks for the comments so far, because they stirred my thinking and motivated fresh ideas, some of which I hope will prove helpful and worth discussing, if and when they mature.

George K.
--
You received this message because you are subscribed to the Google Groups "Everything List" group.
To unsubscribe from this group and stop receiving emails from it, send an email to everything-li...@googlegroups.com.

Alan Grayson

unread,
Apr 20, 2022, 8:14:31 PM4/20/22
to Everything List
On Wednesday, April 20, 2022 at 5:21:47 PM UTC-6 Alan Grayson wrote:


On Friday, April 15, 2022 at 12:41:03 PM UTC-6 meeke...@gmail.com wrote:


On 4/14/2022 2:00 PM, George Kahrimanis wrote:
On Wednesday, April 13, 2022 at 8:55:48 PM UTC+3 meeke...@gmail.com (Brent) wrote:

Decoherence has gone part way in solving the when/where/what basis questions, but only part way.

As I wrote at the end of my first reply to your message, I share your concern about decoherence but I see the glass as half-full; that is, with a little more subtlety I hope that the matter can be formulated in clear terms.

Surely collapse is easier to handle as a general concept (except, on the other hand, that it requires new dynamics). I forgot to mention that my argument for deriving the Born Rule works with collapse, too -- so it is an alternative to Gleason's theorem.

Here I define colapse as an irreversible process, violating unitarity of course, and I keep it separate from randomisation. The latter means that each outcome is somehow randomised -- an assumption we can do without.

Collapse can also be described in a many-world formulation! It differs from the no-collapse MWI only in being irreversible.

If you can throw away low probability branches, what's to stop you from throwing away all but one?  You've already broken unitary evolution.  If you read Hardy's axiomatization of QM you see that the difference between QM and classical mechanics turns on a single word in Axiom 5 Continuity: There exists a continuous reversible transformation on a system between any two pure states of that system.

My argument in outline is
1. assessment that MWI-with-collapse is workable;
2. therefore, outcomes of small enough measure can be neglected in practice;

Yes, I've wondered if a smallest non-zero probability could be defined consistent with the data.

3. now Everett's argument can proceed, concluding that the Born Rule is a practically safe assumption (to put it briefly).

So I have replaced two assumptions of Gleason's theorem, randomisation and non-contextuality, by the assessment of workability only.

If you don't feel comfortable yet with formulating collapse in a many-world setting, let us also assume randomisation (God plays dice), for the sake of the argument, in a single-world formulation. That is, we ASSUME the existence of probability; then the previous argument just guarantees that this probability follows the Born Rule.

Assume?  Randomness is well motivated by evidence.  And it's more random than just not knowing some inherent variable, because in the EPR experiment a randomized hidden variable can on explain the QM result if it's non-local.



Of course I favour the first version of the argument, using the many-world formulation of collapse, to avoid the "God plays dice" nightmare.

Why this fear of true randomness?  We have all kinds of classical randomness we just attributed to "historical accident".  Would it really make any difference it were due to inherent quantum randomness?  Albrect and Phillips have made an argument that there is quantum randomness even nominally classical dynamics. https://arxiv.org/abs/1212.0953v3

The authors regard quantum fluctuations as fundamental. How are they defined? AG
I think I get it. Whereas before QM we could attribute single, unpredicted outcomes to ignorance of initial conditions, and but with QM our understanding is augmented; now we can attribute it to ... nothing? AG

Alan Grayson

unread,
Apr 20, 2022, 9:42:22 PM4/20/22
to Everything List
On Wednesday, April 20, 2022 at 6:14:31 PM UTC-6 Alan Grayson wrote:
On Wednesday, April 20, 2022 at 5:21:47 PM UTC-6 Alan Grayson wrote:


On Friday, April 15, 2022 at 12:41:03 PM UTC-6 meeke...@gmail.com wrote:


On 4/14/2022 2:00 PM, George Kahrimanis wrote:
On Wednesday, April 13, 2022 at 8:55:48 PM UTC+3 meeke...@gmail.com (Brent) wrote:

Decoherence has gone part way in solving the when/where/what basis questions, but only part way.

As I wrote at the end of my first reply to your message, I share your concern about decoherence but I see the glass as half-full; that is, with a little more subtlety I hope that the matter can be formulated in clear terms.

Surely collapse is easier to handle as a general concept (except, on the other hand, that it requires new dynamics). I forgot to mention that my argument for deriving the Born Rule works with collapse, too -- so it is an alternative to Gleason's theorem.

Here I define colapse as an irreversible process, violating unitarity of course, and I keep it separate from randomisation. The latter means that each outcome is somehow randomised -- an assumption we can do without.

Collapse can also be described in a many-world formulation! It differs from the no-collapse MWI only in being irreversible.

If you can throw away low probability branches, what's to stop you from throwing away all but one?  You've already broken unitary evolution.  If you read Hardy's axiomatization of QM you see that the difference between QM and classical mechanics turns on a single word in Axiom 5 Continuity: There exists a continuous reversible transformation on a system between any two pure states of that system.

My argument in outline is
1. assessment that MWI-with-collapse is workable;
2. therefore, outcomes of small enough measure can be neglected in practice;

Yes, I've wondered if a smallest non-zero probability could be defined consistent with the data.

3. now Everett's argument can proceed, concluding that the Born Rule is a practically safe assumption (to put it briefly).

So I have replaced two assumptions of Gleason's theorem, randomisation and non-contextuality, by the assessment of workability only.

If you don't feel comfortable yet with formulating collapse in a many-world setting, let us also assume randomisation (God plays dice), for the sake of the argument, in a single-world formulation. That is, we ASSUME the existence of probability; then the previous argument just guarantees that this probability follows the Born Rule.

Assume?  Randomness is well motivated by evidence.  And it's more random than just not knowing some inherent variable, because in the EPR experiment a randomized hidden variable can on explain the QM result if it's non-local.



Of course I favour the first version of the argument, using the many-world formulation of collapse, to avoid the "God plays dice" nightmare.

Why this fear of true randomness?  We have all kinds of classical randomness we just attributed to "historical accident".  Would it really make any difference it were due to inherent quantum randomness?  Albrect and Phillips have made an argument that there is quantum randomness even nominally classical dynamics. https://arxiv.org/abs/1212.0953v3

The authors regard quantum fluctuations as fundamental. How are they defined? AG
I think I get it. Whereas before QM we could attribute single, unpredictABLE outcomes to ignorance of initial conditions, and but with QM our understanding is augmented; now we can attribute it to ... nothing? AG
Is that because, if we could attribute a single, unpredictable outccome to ignorance, that would be, defacto, a hidden variable theory? AG 

Brent


Thanks for the comments so far, because they stirred my thinking and motivated fresh ideas, some of which I hope will prove helpful and worth discussing, if and when they mature.

George K.
You received this message because you are subscribed to the Google Groups "Everything List" group.
To unsubscribe from this group and stop receiving emails from it, send an email to everything-li...@googlegroups.com.

spudb...@aol.com

unread,
Apr 20, 2022, 9:45:41 PM4/20/22
to johnk...@gmail.com, everyth...@googlegroups.com
Ah, Bohminan pilot wave mechanics. It's probably just part of the Root Kit of the universe? You know, as Wigner's Friend chatted in Mandarin through the slot in Searle's Room, to Schrodinger's Cat, "I knew instantly what you were thinking"

This bon mot, rolled em in the aisles in the faculty lounge at Princeton U. or was it Teaneck? 


--
You received this message because you are subscribed to the Google Groups "Everything List" group.
To unsubscribe from this group and stop receiving emails from it, send an email to everything-li...@googlegroups.com.
To view this discussion on the web visit

Brent Meeker

unread,
Apr 20, 2022, 9:53:33 PM4/20/22
to everyth...@googlegroups.com
Roughtly, yes.  That's what a hidden variable is, a value that if you knew it you could predict the outcome.

Brent

George Kahrimanis

unread,
Apr 21, 2022, 6:03:39 PM4/21/22
to Everything List
In my current way of thinking, the disagreement between Alan Grayson and John K. Clark is about two subtly different concepts under the same name, "probability". For example, when I read "80% chance of rain today", I may think that in some possible futures it will not rain (so probability is meaningless), yet I feel an instinctive urge for protection from bad weather, so I take my umbrella. We are programmed to act in this way, due to Darwinian selection -- but it is a different matter to claim that QM (without collapse) issues a probability for each possible outcome so that then we are rationally obliged to apply Maximisation of Expected Utility. I grant the former but not the latter.

Part of the trouble is that serious philosophical issues about probability are still debated, so that there are traps for anyone who deals with these things. Here is an example.

> [...] until Alan Grayson sees the end of the race, or somebody tells Alan Grayson about it, Alan Grayson can't be certain what world Alan Grayson is in. Alan Grayson could be in a world where horse X won or Alan Grayson could be in a world where horse Y won, until Alan Grayson receives more information Alan Grayson would have to say the odds are 50-50.

If you mean that on sheer ignorance the odds are 50-50, we need some clarifications. Strictly speaking, zero information implies "undefined probability", or "imprecise probability between 0 and 1". The reason it is commonly mistaken as 50-50 is an implied strategy, flipping a coin in case of ignorance, but then the odds are of the coin instead of the object of the bet. (This strategy works only if the agent is free to choose which side of the bet she underwrites.)

For the instrumentalists among us (glad to have you, BTW): the question of interest to me is not about which way is best to derive probability from QM -- that would be a pointless discussion, I agree! The question is whether all of them beg the question, so that we have to think of a rational decision theory without probability.

Although Everett's argument (whose improvement I have proposed) grants that in the long run (that is, large samples) the Born Rule is practically certain to apply, this is not technically the same as probability for each single outcome -- though I admit that it works the same, to trigger an instinctive impulse. But for a RATIONAL decision theory this probability is not granted, IMO.

I can give examples of a decision theory w/o probability, but they would dilute the focus of this message.

George K.

Brent Meeker

unread,
Apr 21, 2022, 6:33:46 PM4/21/22
to everyth...@googlegroups.com


On 4/21/2022 3:03 PM, George Kahrimanis wrote:
In my current way of thinking, the disagreement between Alan Grayson and John K. Clark is about two subtly different concepts under the same name, "probability". For example, when I read "80% chance of rain today", I may think that in some possible futures it will not rain (so probability is meaningless), yet I feel an instinctive urge for protection from bad weather, so I take my umbrella. We are programmed to act in this way, due to Darwinian selection -- but it is a different matter to claim that QM (without collapse) issues a probability for each possible outcome so that then we are rationally obliged to apply Maximisation of Expected Utility. I grant the former but not the latter.

Part of the trouble is that serious philosophical issues about probability are still debated, so that there are traps for anyone who deals with these things. Here is an example.

> [...] until Alan Grayson sees the end of the race, or somebody tells Alan Grayson about it, Alan Grayson can't be certain what world Alan Grayson is in. Alan Grayson could be in a world where horse X won or Alan Grayson could be in a world where horse Y won, until Alan Grayson receives more information Alan Grayson would have to say the odds are 50-50.

If you mean that on sheer ignorance the odds are 50-50, we need some clarifications. Strictly speaking, zero information implies "undefined probability", or "imprecise probability between 0 and 1". The reason it is commonly mistaken as 50-50 is an implied strategy, flipping a coin in case of ignorance, but then the odds are of the coin instead of the object of the bet. (This strategy works only if the agent is free to choose which side of the bet she underwrites.)

If the odds 50/50 can apply to the coin...because you don't know which way it will come down...then the same concept applies to the horse race.



For the instrumentalists among us (glad to have you, BTW): the question of interest to me is not about which way is best to derive probability from QM -- that would be a pointless discussion, I agree! The question is whether all of them beg the question, so that we have to think of a rational decision theory without probability.

Rational decision theory only exists because of uncertainty.  If there were no uncertainty one wouldn't need theory to inform your choice, you would directly by value.

Brent


Although Everett's argument (whose improvement I have proposed) grants that in the long run (that is, large samples) the Born Rule is practically certain to apply, this is not technically the same as probability for each single outcome -- though I admit that it works the same, to trigger an instinctive impulse. But for a RATIONAL decision theory this probability is not granted, IMO.

I can give examples of a decision theory w/o probability, but they would dilute the focus of this message.

George K. --
You received this message because you are subscribed to the Google Groups "Everything List" group.
To unsubscribe from this group and stop receiving emails from it, send an email to everything-li...@googlegroups.com.

Alan Grayson

unread,
Apr 21, 2022, 8:49:09 PM4/21/22
to Everything List
Why the quaified "yes"? Does Bell's theorem exclude ignorance as a hidden variable? AG

Brent Meeker

unread,
Apr 21, 2022, 9:20:50 PM4/21/22
to everyth...@googlegroups.com
Qualified because it's not clear what you mean by "attribute a single, unpredictable outcome to ignorance."  You could attribute a single unpredictable outcome to a cosmic ray hitting your instrument, but that's not an example of a hidden variable, it's just your ignorance of cosmic rays that are incoming.


Does Bell's theorem exclude ignorance as a hidden variable? AG

Ignorance is a constant.

Brent

--
You received this message because you are subscribed to the Google Groups "Everything List" group.
To unsubscribe from this group and stop receiving emails from it, send an email to everything-li...@googlegroups.com.

John Clark

unread,
Apr 22, 2022, 7:13:43 AM4/22/22
to 'Brent Meeker' via Everything List
On Thu, Apr 21, 2022 at 6:04 PM George Kahrimanis <geka...@gmail.com> wrote:

>> [...] until Alan Grayson sees the end of the race, or somebody tells Alan Grayson about it, Alan Grayson can't be certain what world Alan Grayson is in. Alan Grayson could be in a world where horse X won or Alan Grayson could be in a world where horse Y won, until Alan Grayson receives more information Alan Grayson would have to say the odds are 50-50.

> If you mean that on sheer ignorance the odds are 50-50, we need some clarifications.

If quantum mechanic says a photon must be either horizontally or vertically polarized, and it can provide no reason to favor one outcome over the other, then the odds are 50-50. Many Worlds would say that when the photon encounters the polarizer the universe splits into two, one universe contains a George Kahrimanis who sees a photon emerge from a polarizer oriented in the horizontal direction, and one universe contains a George Kahrimanis who does not see the photon emerge and concludes that before it hit the polarizer the photon must've been polarized in the vertical direction and was then destroyed.
 
> Strictly speaking, zero information implies "undefined probability",

Sure, but thanks to quantum mechanics we are not completely clueless about what will happen when a photon of unknown polarization encounters a polarizer oriented in the horizontal direction, we can't be certain of the outcome but we can be certain of certain outcomes that are not possible, and we can obtain probabilities that are very useful about outcomes that ARE possible.  


> For the instrumentalists among us (glad to have you, BTW): the question of interest to me is not about which way is best to derive probability from QM -- that would be a pointless discussion,

It would be pointless because we have known from experiment for nearly a century that the best way to obtain probability from quantum mechanics is to take the square of the absolute value of a particle's wave-function, a.k.a. the Born rule.   

>The question is whether all of them beg the question, so that we have to think of a rational decision theory without probability.

Even in the days before quantum mechanics, as soon as physicists started thinking about thermodynamics they knew that a rational decision theory without probability was not viable.
 
> Although Everett's argument (whose improvement I have proposed) grants that in the long run (that is, large samples) the Born Rule is practically certain to apply, this is not technically the same as probability for each single outcome -- though I admit that it works the same,

I would argue that if X works the same as Y then technically X is Y. 

>  for a RATIONAL decision theory this probability is not granted,

IF that's true THEN a RATIONAL man will consistently make predictions about the outcome of an experiment that are inferior to the predictions that an IRRATIONAL man would make. So there would be no point to rationality or being "rational". THEREFORE I conclude that your above statement is not true.

John K Clark    See what's on my new list at  Extropolis
pbc

arq



 

Alan Grayson

unread,
Apr 22, 2022, 11:07:36 AM4/22/22
to Everything List
I think the general assumption is that ignorance refers to measurement errors. They can be systematic and thus constant. Does Bell prohibitions against hidden variables apply to this situation? AG 

Brent Meeker

unread,
Apr 22, 2022, 12:00:47 PM4/22/22
to everyth...@googlegroups.com
Measurement is just a context, not a kind, of error.  The cosmic ray could produce an error in a measurement.  You're thinking of bias errors, which are those which whose mean is non-zero.  But that's just a statistical category; it doesn't say anything about the source.  A hidden variable source of uncertainty is one that in principle you could know and eliminate the uncertainty...that's why it's called "hidden".  So it is implicitly uncertainty due to ignorance, not in general, but specific ignorance of the value of the hidden variable.

Brent

AG 


Does Bell's theorem exclude ignorance as a hidden variable? AG

Ignorance is a constant.

Brent

--
You received this message because you are subscribed to the Google Groups "Everything List" group.
To unsubscribe from this group and stop receiving emails from it, send an email to everything-li...@googlegroups.com.
--
You received this message because you are subscribed to the Google Groups "Everything List" group.
To unsubscribe from this group and stop receiving emails from it, send an email to everything-li...@googlegroups.com.

George Kahrimanis

unread,
Apr 22, 2022, 7:00:48 PM4/22/22
to Everything List
On Friday, April 22, 2022 at 1:33:46 AM UTC+3 meeke...@gmail.com wrote:
On 4/21/2022 3:03 PM, George Kahrimanis wrote:
[...] Strictly speaking, zero information implies "undefined probability", or "imprecise probability between 0 and 1". The reason it is commonly mistaken as 50-50 is an implied strategy, flipping a coin in case of ignorance, but then the odds are of the coin instead of the object of the bet. (This strategy works only if the agent is free to choose which side of the bet she underwrites.)

If the odds 50/50 can apply to the coin...because you don't know which way it will come down...then the same concept applies to the horse race.

No, I do have information about this coin: I have tossed it many times. I am clueless about this horserace. Big difference. Concentrate!

[...]  we have to think of a rational decision theory without probability.
Rational decision theory only exists because of uncertainty.  If there were no uncertainty one wouldn't need theory to inform your choice, you would directly by value.

Now you are justified to be buffled, because I have avoided giving any example. Here is one, containing a combination of uncertainties and certainties -- the latter are "moral certainties", something like "FAPP" but well defined.

Example. When I have a choice between acting recklessly and acting carefully, and my spirit of adventure overcomes my instict of survival, a rational argument IMO is to think of my insurance: they will increase the premium or drop me if they classify me as a reckless man. I need insurance because of uncertainty, to protect my future selves as well as my loved ones in future branches in which I will not exist. To keep the example short, I postpone arguing why insurance provides moral certainty (in principle).

George K.

George Kahrimanis

unread,
Apr 22, 2022, 8:49:27 PM4/22/22
to Everything List
On Friday, April 22, 2022 at 2:13:43 PM UTC+3 johnk...@gmail.com wrote:
On Thu, Apr 21, 2022 at 6:04 PM George Kahrimanis <geka...@gmail.com> wrote:
 
> Strictly speaking, zero information implies "undefined probability",

Sure, but[...]

Sorry, but if it is undefined then there is no "but". You remind me of myself a few decades ago, when I was in love with Bayesian inference and I defended the use of artificial priors. Later I understood that BI is just a heuristic method, not sound inference (unless the prior is true). A flat prior, or probabilities 1/2 in this case, are only tools in a heuristisc method, not proper descriptions of ignorance.

Anyway, my point in this example was only that probability is a very tricky subject.

> For the instrumentalists among us (glad to have you, BTW): the question of interest to me is not about which way is best to derive probability from QM -- that would be a pointless discussion,

It would be pointless because we have known from experiment for nearly a century that the best way to obtain probability from quantum mechanics is to take the square of the absolute value of a particle's wave-function, a.k.a. the Born rule.

Not only I do not argue with this, but I emphasise it: this is one of the ways in which QM appears "workable". But my point was to prepare the reader for a version of the Born rule concerning large samples only, instead of single outcomes. Surely the experimental evidence is from large samples; the probability for a single case is an extrapolation which is "a matter of course" for a certain way of thinking, but not technically obligatory.
 
>The question is whether all of them beg the question, so that we have to think of a rational decision theory without probability.

Even in the days before quantum mechanics, as soon as physicists started thinking about thermodynamics they knew that a rational decision theory without probability was not viable.

In my anwer to Brent (my previous message) I gave an example.

> Although Everett's argument (whose improvement I have proposed) grants that in the long run (that is, large samples) the Born Rule is practically certain to apply, this is not technically the same as probability for each single outcome -- though I admit that it works the same,

I would argue that if X works the same as Y then technically X is Y.

Careful! You trimmed off the end of my sentence: "... it works the same, to trigger an instinctive impulse".  Sorry for my sloppy syntax: I meant "it works the same, with regard to triggering an instinctive impulse". Noy always, not necessarily.

Instead of "technically" you should have "practically", in the sense "a technical distinction without a practical difference".

>  for a RATIONAL decision theory this probability is not granted,

IF that's true THEN a RATIONAL man will consistently make predictions about the outcome of an experiment that are inferior to the predictions that an IRRATIONAL man would make. So there would be no point to rationality or being "rational". THEREFORE I conclude that your above statement is not true.

(I emphasised "rational" as opposed to an experimentally derived decision theory.)

A good point, but I am an inadequate amateur in the subject you bring up. Surely it is more fun to be irrational, for a while at least, and we all do it. Besides, there is no point in being rational about taking an umbrella in the morrning, unless there are grave consequences to reckon with. Moreover, rationality is about organising certain basic irrational pursuits, typically thinkgs like security, food, sex, and entertainment; priorities are for to the agent to define. Not a black-or-white dichotomy, therefore.
 
And I did not say that there are rational versus irrational predictions. My concern is about the interpretation of probability for a single outcome. If it is a matter of pleasure without any worrying consequences, then the irrational interpretation is fine, even from the rational point of view,

At least we agree on the MWI! The other issues will be resolved, I hope.

George K.

John Clark

unread,
Apr 23, 2022, 8:08:43 AM4/23/22
to 'Brent Meeker' via Everything List
On Fri, Apr 22, 2022 at 8:49 PM George Kahrimanis <geka...@gmail.com> wrote:

>>> Strictly speaking, zero information implies "undefined probability",

>> Sure, but [...]

> Sorry, but if it is undefined then there is no "but".

In this case there is a "but" because thanks to quantum mechanics there is NOT zero information about what will happen when a photon of unknown polarization encounters a polarizer oriented in the horizontal direction; quantum mechanics says there are 2 and only 2 possible outcomes to that situation and it also says there is no reason to favor one of those outcomes over the other

> my point was to prepare the reader for a version of the Born rule concerning large samples only, instead of single outcomes.

Such a version would be less powerful and less useful because if a gambler wished to make money he would be foolish to ignore the Born rule when placing bets or setting odds, even for one time events. So there doesn't seem to be much point in developing such a version.   

>>> Although Everett's argument (whose improvement I have proposed) grants that in the long run (that is, large samples) the Born Rule is practically certain to apply, this is not technically the same as probability for each single outcome -- though I admit that it works the same,

>> I would argue that if X works the same as Y then technically X is Y.

> Careful! You trimmed off the end of my sentence: "... it works the same, to trigger an instinctive impulse". 

I trimmed your sentence because it was redundant. As I said if X works the same as Y then technically X is Y, so obviously X would trigger the same instinctive impulse that Y does, and X would trigger everything else that Y triggers too because X works the same as Y

If X works the same as Y then how does it still make sense to say that  X is not Y?

> Sorry for my sloppy syntax: I meant "it works the same, with regard to triggering an instinctive impulse". Noy always, not necessarily.
 
If the use of probability, even for single occurrences, triggers an instinctive impulse then it must've conferred an evolutionary advantage over people in which that instinctive impulse was lacking, so the use of probability, even for single occurrences, must confer a survival advantage because Evolution cannot be fooled by philosophical bafflegab. If a rational person were forced to play Russian roulette (a one time event) but was given a choice between using a revolver that had 1 bullet and 5 empty chambers and a revolver that had 5 bullets and 1 empty chamber, which revolver would a rational person choose?

> Instead of "technically" you should have "practically", in the sense "a technical distinction without a practical difference".

I used that word because you originally said  "the Born Rule is practically certain to apply, this is not technically the same as probability for each single outcome -- though I admit that it works the same".


>>>  for a RATIONAL decision theory this probability is not granted,

>> IF that's true THEN a RATIONAL man will consistently make predictions about the outcome of an experiment that are inferior to the predictions that an IRRATIONAL man would make. So there would be no point to rationality or being "rational". THEREFORE I conclude that your above statement is not true.

> (I emphasised "rational" as opposed to an experimentally derived decision theory.)

I can't think of anything more rational than basing your ideas about how the world works on observation and experimentation. Aristotle wrote that women had fewer teeth than men, it's known that he was married, twice in fact, yet he never thought of just looking into his wife's mouth and counting. Aristotle thought that just by sitting and thinking he could figure out how the world works. Aristotle was not being rational.

 
> Moreover, rationality is about organising certain basic irrational pursuits, typically thinkgs like security, food, sex, and entertainment; priorities are for to the agent to define.

Entertainment perhaps but, going back for billions of years, if every single one of your ancestors, without exception, was not obsessed with security, food and sex, you would not exist. And the same could be said about every other organism on the planet  
 
> Not a black-or-white dichotomy, therefore.

I think it is a black-or-white dichotomy, an organism can get its genes into the next generation or it cannot 

> Example. When I have a choice between acting recklessly and acting carefully, and my spirit of adventure overcomes my instict of survival, a rational argument IMO is to think of my insurance: they will increase the premium or drop me if they classify me as a reckless man. I need insurance because of uncertainty, to protect my future selves as well as my loved ones in future branches in which I will not exist.

In Evolution there is always a trade-off, being too courageous can reduce your chances of getting your genes into the next generation, but being too cowardly can too. If I'm too courageous I might get in a fight I have no chance of winning but if I'm too cowardly and refuse to take any risks I might starve to death. The best adapted organism for survival would be one in the middle of the brave-coward bell curve but admires those on the far edge of the curve who have more courage than they do and likes to hang around with them; because then you can let them take the lead attacking the gigantic woolly mammoth while you play a secondary role, be brave enough to make it clear that your participation in the hunt is useful but leave the extreme heroics to others. If the hero is successful in the hunt he can't eat an entire Mammoth by himself so you get some of the leftovers. If he's unsuccessful and gets killed then find somebody else who's braver than you, hang around with them, and hope he has better luck.

John K Clark    See what's on my new list at  Extropolis
wmm




 

Alan Grayson

unread,
Apr 23, 2022, 11:05:46 AM4/23/22
to Everything List
On Tuesday, April 19, 2022 at 2:43:47 PM UTC-6 johnk...@gmail.com wrote:
If you don't know that the SE is time DEPENDENT, at least one of its forms, you should refrain from posing as a expert on its interpretation. Further, in the case of a free particle, the solution changes its form as tIme goes backward, or forward, so your comment shows ignorance of what time dependence means. AG 

John Clark

unread,
Apr 23, 2022, 1:45:08 PM4/23/22
to 'Brent Meeker' via Everything List
On Sat, Apr 23, 2022 at 11:05 AM Alan Grayson <agrays...@gmail.com> wrote:

>>>> Schrödinger's Equation is time independent,

>>> Then why, for example, does the solution for a free particle spread out as time progresses? AG 

>> As time progresses things change, that is in fact what time means. So if something spreads out as time progresses if you reverse time then that "something" would converge. Schrodinger's wave equation works in either direction, no information is lost so if you know what the wave looks like now you can figure out what it will look like tomorrow and also figure out what it looked like yesterday.   

> If you don't know that the SE is time DEPENDENT, at least one of its forms, you should refrain from posing as a expert on its interpretation

You've forgotten how all this started, you said "but S's equation just gives the time dependent probabilities BEFORE a measurement is taken" , and I made it clear that Schrödinger's Equation is independent of if time is going forwards or backwards, so if you know what the quantum wave of a particle is today the day after a measurement has been taken then Schrödinger's Equation can tell you what the quantum wave will be tomorrow, and also what the quantum wave was the day before yesterday, the day before a measurement will be taken. This is my exact quote and I still stand by every word of it:

"Schrödinger's Equation is time independent, it works just as well forwards or backwards, so "before" or "after" are irrelevant terms. And Schrödinger makes no use of "measurement" and says nothing about it".

So there are only two conclusions possible, either Schrödinger's Equation is just wrong and needs to be drastically modified, or Many Worlds is correct. I think Schrödinger's Equation works pretty well just as it is. 

By the way, Erwin Schrödinger made no secret of being a sexual libertine, but now his politically incorrect lifestyle is catching up with him, there is a move afoot by the same sort of imbeciles who dreamed up the phrase "defund the police" to change one thing in the equation, its name. They're also trying to change the name of the James Webb telescope.  

> Further, in the case of a free particle, the solution changes its form as tIme goes backward,

Of course the solutions change depending on if time is going forwards or backwards! If it didn't it wouldn't conform with reality and would be absolutely useless because since the days of Ogg the caveman humanity has known that yesterday was different than today and feels very confident that tomorrow will be different from today.   

>  your comment shows ignorance of what time dependence means. AG 

At least I'm not so ignorant as to think that we've been putting things into orbit for nearly 70 years without obtaining hypersonic speed, or that flying saucer men landed in Roswell New Mexico in 1948. And I never claimed, as you have, that you were the co-author of a scientific paper with Carl Sagan sometime in the 1960's but have completely forgotten the exact date of the paper, the journal the paper was printed in, the topic of the paper, and even the name you were using back then; and if anybody on this list believes that then there's a bridge I'd like to sell you.

John K Clark    See what's on my new list at  Extropolis
ews



Alan Grayson

unread,
Apr 23, 2022, 2:35:18 PM4/23/22
to Everything List
On Saturday, April 23, 2022 at 11:45:08 AM UTC-6 johnk...@gmail.com wrote:
On Sat, Apr 23, 2022 at 11:05 AM Alan Grayson <agrays...@gmail.com> wrote:

>>>> Schrödinger's Equation is time independent,

>>> Then why, for example, does the solution for a free particle spread out as time progresses? AG 

>> As time progresses things change, that is in fact what time means. So if something spreads out as time progresses if you reverse time then that "something" would converge. Schrodinger's wave equation works in either direction, no information is lost so if you know what the wave looks like now you can figure out what it will look like tomorrow and also figure out what it looked like yesterday.   

> If you don't know that the SE is time DEPENDENT, at least one of its forms, you should refrain from posing as a expert on its interpretation

You've forgotten how all this started, you said "but S's equation just gives the time dependent probabilities BEFORE a measurement is taken" , and I made it clear that Schrödinger's Equation is independent of if time is going forwards or backwards, so if you know what the quantum wave of a particle is today the day after a measurement has been taken then Schrödinger's Equation can tell you what the quantum wave will be tomorrow, and also what the quantum wave was the day before yesterday, the day before a measurement will be taken. This is my exact quote and I still stand by every word of it:

"Schrödinger's Equation is time independent, it works just as well forwards or backwards, so "before" or "after" are irrelevant terms. And Schrödinger makes no use of "measurement" and says nothing about it".

So there are only two conclusions possible, either Schrödinger's Equation is just wrong and needs to be drastically modified, or Many Worlds is correct. I think Schrödinger's Equation works pretty well just as it is. 


The advantage for you is your use of a private language. In the SE, I see d/dt. Ergo, the equation is time dependent. I am not impressed or interested in your self-serving obscurations.

As for the Sagan issue, as I distinctly recall that I posted the citations to those articles, one with Sagan and one of his doctoral students at the time at Harvard, David Morrison (check him out on Wiki), and a second paper. possibly just with Sagan or just with Morrison, in 1967 or 1968. And No, I wasn't at Harvard, but working at the Smithsonian Astrophysical Observatory, located at 67 Garden Street, in Cambridge MA. One or both were printed in The Astronomical Journal. I also told you that my real name is NOT Grayson, but apparently you couldn't connect the dots. 

As for Roswell, for someone who firmly believes in the most fantastical and improbable interpretation of QM, I don't see that you're in a position to cast aspersions on my belief in the Roswell Incident. What do you think the US Navy pilots were chasing fairly recently? Do you have a clue, or do you just like to demonstrate how closed you are to unusual phenomena? You regard yourself as an objective analyst of scientific facts, but you're anything but; just another fool shooting off your mouth.

AG

John Clark

unread,
Apr 23, 2022, 3:40:16 PM4/23/22
to 'Brent Meeker' via Everything List
On Sat, Apr 23, 2022 at 2:35 PM Alan Grayson <agrays...@gmail.com> wrote:

> As for the Sagan issue, as I distinctly recall that I posted the citations to those articles,

I distinctly remember asking you over and over and over for a link to the journal articles (at least 2 you said) that you claim to have written with Sagan, BUT I most certainly do NOT remember you actually doing so. But it's easy enough to settle this issue, just post a link to these wonderful papers you claim to have written with Sagan right now.
 
> and a second paper. possibly just with Sagan or just with Morrison, in 1967 or 1968.

You can't seem to get your stories straight. So now you can't even remember who you wrote the paper with in addition to not remembering the topic of the paper, nor exactly where or when it was published. It must've been a very forgettable paper. 

> David Morrison (check him out on Wiki)

I'm sure Wikipedia is the place where you found the name "David Morrison", and I'm sure he wrote a paper with Carl Sagan. I am also sure that Carl Sagan would not have wanted to co-write an astrophysical journal paper with somebody who didn't even know you needed to achieve hypersonic speed to get into orbit. 

> And No, I wasn't at Harvard,

What a surprise!  
 
> but working at the Smithsonian Astrophysical Observatory, located at 67 Garden Street, in Cambridge MA.

Wow, you must be telling the truth because it would be impossible for you to know what the address of the Smithsonian Astrophysical Observatory was unless you once worked there, as it is well known that the Smithsonian keeps its address top-secret, tourists only find it by blindly stumbling around and coming across it by sheer chance.

> One or both were printed in The Astronomical Journal. I also told you that my real name is NOT Grayson,

And my real name is not John K Clark but is Albert Einstein, I'm getting a little old these days but I've published all sorts of wonderful scientific papers, although I can't now remember the topics of a single one of them.
 
Mr. Mystery Man, if Alan Grayson is not your real name then you, whoever you are, have presented absolutely no evidence, much less proof, that you ever wrote a paper with Carl Sagan or with anybody else, zero zilch nada goose egg. But you have inadvertently presented plenty of circumstantial evidence that you could not possibly have.

> As for Roswell, for someone who firmly believes in the most fantastical and improbable interpretation of QM,

If I'm a fool for believeing in Many Worlds then I'm in good company because among physicists it's the second most popular quantum interpretation, right after shut up and calculate.  

> Do you have a clue, or do you just like to demonstrate how closed you are to unusual phenomena?

Yep, I'm communicating with a very unusual phenomenon right now.  

John K Clark    See what's on my new list at  Extropolis
vyn

vup

Alan Grayson

unread,
Apr 23, 2022, 4:17:00 PM4/23/22
to Everything List
You're a sad case, indeed. Those two papers were published around 54 years ago, so their references aren't at my fingertips. The titles were, "The Martian Wave of Darkening and Related Phenomena", and "Hypersensitization of Infrared Sensitive Plates (or Emulsions)". I recall that Carl really liked this second paper, presumably because in those days it was hard to observe anything planetary in great detail. We increased the sensitivity by a factor of 10, possibly more. The first was with Sagan and Morrison, and the second was just with Morrison, IIRC. The first was published in The Astronomical Journal, and I can't recall offhand where the second was published. When I gave you the address of the SAO, I internally predicted your dumb response and wasn't surprised. And Yes, you're a fool for believing in MWI, apparently because other fools share your belief. But MANY prominent scientists do NOT share that belief, such as the late Steven Weinberg. BTW, if the energy of other worlds depends on their probability of occurring, as Sean Carroll alleges, does gravity remain the same while the bridges don't collapse, or do the cars using them mysteriously lose mass according to Born's rule? AG
Message has been deleted

John Clark

unread,
Apr 23, 2022, 5:04:08 PM4/23/22
to 'Brent Meeker' via Everything List


On Sat, Apr 23, 2022 at 4:17 PM Alan Grayson <agrays...@gmail.com> wrot

 
Those two papers were published around 54 years ago, so their references aren't at my fingertips.

They haven't been at your fingertips for a very long time, I first asked for them about  10 years ago when you first made that claim..  

The titles were, "The Martian Wave of Darkening and Related Phenomena",

Almost right , the real title, is  "A statistical analysis of the Martian wave of darkening and related phenomena", so you must be either James B Polack, Edward H Greenberg, or Carl Sagan himself who faked his own death for some reason. 
> and "Hypersensitization of Infrared Sensitive Plates (or Emulsions)".
 
Carl Sagan did not write this paper David Morrison and Edward H Greenberg did, so who the hell is  Alan Grayson? Edward H Greenberg is the only one of these people who seems to have slipped into complete obscurity. There is a Edward H Greenberg who is a gynecologist in Fort Lauderdale Florida, there's a Edward H Greenberg who died in 2010 at the age of 93, there's a Edward H Greenberg who died in 1997 at the age of 87, and there is a a Edward H Greenberg who died in 1967 at the age of 84. That's all even the best search engines on the Internet know about Edward H Greenberg. So which Edward H Greenberg are you?

John K Clark    See what's on my new list at  Extropolis
ghe
v

Alan Grayson

unread,
Apr 23, 2022, 5:22:53 PM4/23/22
to Everything List
Since the 1967 paper was co-authored by Sagan and Pollack, why do you conclude that Sagan faked his death, when that occurred in 1996 (and Pollack in 1994)? You seem obsessed with this issue.  More important is the bridge issue as it effects the MWI. AG

John Clark

unread,
Apr 23, 2022, 5:30:46 PM4/23/22
to 'Brent Meeker' via Everything List
On Sat, Apr 23, 2022 at 5:22 PM Alan Grayson <agrays...@gmail.com> wrote:

> You seem obsessed with this issue.  More important is the bridge issue as it effects the MWI. AG

Yeah, if I had been caught telling a whopper as large as the one you told I'd want to change the subject too.
r4x

Alan Grayson

unread,
Apr 23, 2022, 5:48:03 PM4/23/22
to Everything List
On Saturday, April 23, 2022 at 3:30:46 PM UTC-6 johnk...@gmail.com wrote:
On Sat, Apr 23, 2022 at 5:22 PM Alan Grayson <agrays...@gmail.com> wrote:

> You seem obsessed with this issue.  More important is the bridge issue as it effects the MWI. AG

Yeah, if I had been caught telling a whopper as large as the one you told I'd want to change the subject too.

Suppose I told you I am the obscure one, having faded from my previous glory of working the Great One, Carl Sagan? Would that help? Actually I wasn't that impressed with the Morrison paper, but maybe I should have been! At the time I didn't appreciate the difficulty of getting any physics from those blurry images. AG

John Clark

unread,
Apr 23, 2022, 5:56:47 PM4/23/22
to 'Brent Meeker' via Everything List
On Sat, Apr 23, 2022 at 5:48 PM Alan Grayson <agrays...@gmail.com> wrote:

> Suppose I told you I am the obscure one, having faded from my previous glory of working the Great One, Carl Sagan? Would that help?

It might, if you didn't just tell me but gave me one shred of evidence to indicate you actually are Edward H Greenberg, but you have never even come close to doing so.
 John K Clark    See what's on my new list at  Extropolis
ghe

Alan Grayson

unread,
Apr 23, 2022, 6:09:34 PM4/23/22
to Everything List
What kind of evidence would satisfy you? I'll consider it, but only IF you solve the bridge problem, or failing that admit that Sean's solution is completely delusional. AG 

Alan Grayson

unread,
Apr 24, 2022, 2:18:09 PM4/24/22
to Everything List
I get it. No problem with those bridges. After all, Sean Carroll endorses it and he's on the facuty of Caltech! AG 

John Clark

unread,
Apr 24, 2022, 2:34:34 PM4/24/22
to 'Brent Meeker' via Everything List
On Sun, Apr 24, 2022 at 2:18 PM Alan Grayson <agrays...@gmail.com> wrote:

I get it. No problem with those bridges. After all, Sean Carroll endorses it and he's on the facuty of Caltech! AG 

Well I don't get it, I have no idea what you're talking about

John K Clark    See what's on my new list at  Extropolis
kuq




Alan Grayson

unread,
Apr 24, 2022, 7:12:45 PM4/24/22
to Everything List
On Sunday, April 24, 2022 at 12:34:34 PM UTC-6 johnk...@gmail.com wrote:
On Sun, Apr 24, 2022 at 2:18 PM Alan Grayson <agrays...@gmail.com> wrote:

I get it. No problem with those bridges. After all, Sean Carroll endorses it and he's on the facuty of Caltech! AG 

Well I don't get it, I have no idea what you're talking about

Maybe because you're mentally retarded? You posted Sean's "explanation" for where the energy comes from to create the world's which infatuate you! If a world has 1% probability of existing according to Born's rule, it has 1% of the original total energy! This is pure genius, from Caltech! He must have gotten this from the SE, right? AG 

Bruce Kellett

unread,
Apr 24, 2022, 7:39:46 PM4/24/22
to Everything List
Yes, the idea is complete nuts. If I want to check energy conservation in neutron decay, I compare the mass-energy of the original neutron to the sum of the mass-energies of the decay products plus any kinetic energy of these decay products. I want to do this for a decay in a time period where the probability of decay is 1%. According to the division of energy according to the Born probabilities, my check of energy conservation will fail: I find that the decay products have only 1% of the initial energy. So, far from protecting energy conservation in the MWI splitting process, all that has been achieved is a demonstration that energy is never conserved in any quantum process. As stated, this is nuts!

Bruce

John Clark

unread,
Apr 24, 2022, 8:28:02 PM4/24/22
to 'Brent Meeker' via Everything List
On Sun, Apr 24, 2022 at 7:12 PM Alan Grayson <agrays...@gmail.com> wrote:

> Maybe because you're mentally retarded? You posted Sean's "explanation" for where the energy comes from to create the world's which infatuate you! If a world has 1% probability of existing according to Born's rule, it has 1% of the original total energy!

I've explained this to you before but that time I used words that an intelligent adult should understand, but you didn't, so this time I'll imagine I'm speaking to a child with a learning disability, maybe that will work. We've known for a long time there's no way to detect the absolute energy level of anything, we can only detect the energy difference between two things, but there is no way an observer in one universe can compare his energy level with an observer in another universe, so the fact that one universe may have 10 times more energy than another has no observable consequences to anybody in either universe.

> This is pure genius, from Caltech!

Any professor of theoretical physics at Caltech is one hell of a lot smarter than you Mr. Carl Sagan co-author, Mr. Flying-Saucer-Men-Landed-In-Roswell-New-Mexico, And unlike you I'm damn sure he knows enough grade school physics to understand that you need to obtain hyper sonic speed to get into Earth orbit. 

 John K Clark    See what's on my new list at  Extropolis
fgm


Bruce Kellett

unread,
Apr 24, 2022, 9:09:23 PM4/24/22
to Everything List
On Mon, Apr 25, 2022 at 10:28 AM John Clark <johnk...@gmail.com> wrote:
On Sun, Apr 24, 2022 at 7:12 PM Alan Grayson <agrays...@gmail.com> wrote:

> Maybe because you're mentally retarded? You posted Sean's "explanation" for where the energy comes from to create the world's which infatuate you! If a world has 1% probability of existing according to Born's rule, it has 1% of the original total energy!

I've explained this to you before but that time I used words that an intelligent adult should understand, but you didn't, so this time I'll imagine I'm speaking to a child with a learning disability, maybe that will work. We've known for a long time there's no way to detect the absolute energy level of anything, we can only detect the energy difference between two things, but there is no way an observer in one universe can compare his energy level with an observer in another universe, so the fact that one universe may have 10 times more energy than another has no observable consequences to anybody in either universe.


This is what Sean Carroll actually says in his book "Something Deeply Hidden":

"Well", replied Alice. "Just think about ordinary textbook quantum mechanics. Given a quantum state, we can calculate the total energy it describes. As long as the wave function evolves strictly according to the Schrodinger equation, that energy is conserved, right?" ....
"Not all worlds are created equal. Think about the wave function. When it describes multiple branched worlds, we can calculate the total amount of energy by adding up the amount of energy in each world, times the weight (the amplitude squared) for that world. When one world divides in two, the energy in each world is basically the same as it previously was in the single world (as far as anyone living inside is concerned), but their contributions to the total energy of the wave function of the universe have divided in half, since their amplitudes have decreased. Each world got a bit thinner, although its inhabitants can't tell the difference." (page 173)

In other words, Sean is saying that energy conservation works for the multiverse, and he implies that it also works in each individual branch. This is nonsense --  you can't have both. If energy is conserved over the multiverse, then it cannot be conserved in each branch separately, as my previous example of a neutron decay indicates. Energy conservation is routinely observed and checked in individual branches. No one has ever checked energy conservation in the multiverse.

The idea that this energy is conserved in the multiverse derives from the observation that the Schrodinger equation is time translation invariant. Consequently, there is a definite tension between the application of the Schrodinger equation to obtain a multiverse, Noether's theorem, and the routine observation that energy is conserved separately in each branch. The trouble with Sean's glib response to the question is that in each branch of the multiverse, we can measure the energy both before and after the supposed split. These energies are found to be equal in the branch, so energy cannot be conserved over the multiverse, as Alice in Sean's discussion claims.

Despite Carroll's protestations (and the similar protestations of others), energy cannot be conserved in the multiverse -- each split must duplicate the energy of the whole as many times as there are branches.

Bruce

Alan Grayson

unread,
Apr 24, 2022, 9:14:57 PM4/24/22
to Everything List
On Sunday, April 24, 2022 at 6:28:02 PM UTC-6 johnk...@gmail.com wrote:
On Sun, Apr 24, 2022 at 7:12 PM Alan Grayson <agrays...@gmail.com> wrote:

> Maybe because you're mentally retarded? You posted Sean's "explanation" for where the energy comes from to create the world's which infatuate you! If a world has 1% probability of existing according to Born's rule, it has 1% of the original total energy!

I've explained this to you before but that time I used words that an intelligent adult should understand, but you didn't, so this time I'll imagine I'm speaking to a child with a learning disability, maybe that will work. We've known for a long time there's no way to detect the absolute energy level of anything, we can only detect the energy difference between two things, but there is no way an observer in one universe can compare his energy level with an observer in another universe, so the fact that one universe may have 10 times more energy than another has no observable consequences to anybody in either universe.

Then Sean ought to clean up his language. What he writes is surely consistent with my interpretation. AG 

> This is pure genius, from Caltech!

Any professor of theoretical physics at Caltech is one hell of a lot smarter than you Mr. Carl Sagan co-author, Mr. Flying-Saucer-Men-Landed-In-Roswell-New-Mexico, And unlike you I'm damn sure he knows enough grade school physics to understand that you need to obtain hyper sonic speed to get into Earth orbit. 

I agree that Sean is smarter than me. But what you miss is that at the same time he's considerably dumber than me! 

You keep obsessing about my professional relationship with Sagan, and to what end? I'm the one who faded into (Internet) obscurity, and you have enough information to know that my citations are accurate. So get some maturity and STFU. BTW, it matters nothing to me about your skepticism about the Roswell Incident. Opinions can differ. But what do you think US Navy pilots were pursuing fairly recently? AG 

Alan Grayson

unread,
Apr 25, 2022, 1:32:31 AM4/25/22
to Everything List
On Sunday, April 24, 2022 at 6:28:02 PM UTC-6 johnk...@gmail.com wrote:
I was thinking of level flight at supersonic speed, not escape velocity. 

As for papers with CS, there was a third one, where Carl promised to put my name first. It involved the atmosphere of Jupiter, and a series solution, the convergence of which in closed form that I determined. Over the next year I asked him when he would write it. He was always affirmative but did nothing, which left a lasting negative impression. I would have preferred his candidness; possibly that the numerical results, obtained by Morrison, didn't reveal any interesting physics. I never disclosed my dissatisfaction, and later had him write a letter of recommendation for a position at JPL. I got the job and worked there from 1984 to 1998 on the Galileo Project, eventually becoming the Flight Software Cognizant Engineer. The software never failed, except in very few trivial instances. AG

Alan Grayson

unread,
Apr 25, 2022, 1:48:01 AM4/25/22
to Everything List
On Sunday, April 24, 2022 at 11:32:31 PM UTC-6 Alan Grayson wrote:
On Sunday, April 24, 2022 at 6:28:02 PM UTC-6 johnk...@gmail.com wrote:
On Sun, Apr 24, 2022 at 7:12 PM Alan Grayson <agrays...@gmail.com> wrote:

> Maybe because you're mentally retarded? You posted Sean's "explanation" for where the energy comes from to create the world's which infatuate you! If a world has 1% probability of existing according to Born's rule, it has 1% of the original total energy!

I've explained this to you before but that time I used words that an intelligent adult should understand, but you didn't, so this time I'll imagine I'm speaking to a child with a learning disability, maybe that will work. We've known for a long time there's no way to detect the absolute energy level of anything, we can only detect the energy difference between two things, but there is no way an observer in one universe can compare his energy level with an observer in another universe, so the fact that one universe may have 10 times more energy than another has no observable consequences to anybody in either universe.

> This is pure genius, from Caltech!

Any professor of theoretical physics at Caltech is one hell of a lot smarter than you Mr. Carl Sagan co-author, Mr. Flying-Saucer-Men-Landed-In-Roswell-New-Mexico, And unlike you I'm damn sure he knows enough grade school physics to understand that you need to obtain hyper sonic speed to get into Earth orbit. 

I was thinking of level flight at supersonic speed, not escape velocity. 

As for papers with CS, there was a third one, where Carl promised to put my name first. It involved the atmosphere of Jupiter, and a series solution, the convergence of which in closed form that I determined. Over the next year I asked him when he would write it. He was always affirmative but did nothing, which left a lasting negative impression. I would have preferred his candidness; possibly that the numerical results, obtained by Morrison, didn't reveal any interesting physics. I never disclosed my serious dissatisfaction, and later had him write a letter of recommendation for a position at JPL. I got the job and worked there from 1984 to 1998 on the Galileo Project, eventually becoming the Flight Software Cognizant Engineer. The software never failed, except in very few trivial instances. AG

While I was at JPL, I wrote a 100 page paper on Strategic Arms Control. The Foreign Policy journal wanted to edit and publish it. I should have agreed, but refused the offer, preferring instead to publish it in Foreign Affairs, but that journal showed no interest. That was probably the dumbist thing I ever did. AG

John Clark

unread,
Apr 25, 2022, 5:31:52 AM4/25/22
to 'Brent Meeker' via Everything List
On Sun, Apr 24, 2022 at 9:09 PM Bruce Kellett <bhkel...@gmail.com> wrote:

> This is what Sean Carroll actually says in his book "Something Deeply Hidden":

"Well", replied Alice. "Just think about ordinary textbook quantum mechanics. Given a quantum state, we can calculate the total energy it describes. As long as the wave function evolves strictly according to the Schrodinger equation, that energy is conserved, right?" ....
"Not all worlds are created equal. Think about the wave function. When it describes multiple branched worlds, we can calculate the total amount of energy by adding up the amount of energy in each world, times the weight (the amplitude squared) for that world. When one world divides in two, the energy in each world is basically the same as it previously was in the single world (as far as anyone living inside is concerned), but their contributions to the total energy of the wave function of the universe have divided in half, since their amplitudes have decreased. Each world got a bit thinner, although its inhabitants can't tell the difference." (page 173)

In other words, Sean is saying that energy conservation works for the multiverse, and he implies that it also works in each individual branch. This is nonsense --  you can't have both. If energy is conserved over the multiverse, then it cannot be conserved in each branch separately,

Nonsense.  You can ifas Sean Carroll says you  "add up the amount of energy in each world, times the weight (the amplitude squared) for that world". I think you've forgotten your first year calculus, it's possible to add up an infinite number of numbers and get a finite result.

> Energy conservation is routinely observed and checked in individual branches.

Correct.  

> No one has ever checked energy conservation in the multiverse.

Correct again, there is no experimental confirmation that energy is conserved in the multiverse, and it would violate no law of logic if it was not, but most versions of Many Worlds assume energy is conserved.  And nothing in the above contradicts what Sean Carroll said.  

> The idea that this energy is conserved in the multiverse derives from the observation that the Schrodinger equation is time translation invariant.

With cosmology you can't assume Schrodinger's equation tells you all you need to know because it says absolutely nothing about gravity and doesn't include General Relativity. And we don't even know for a fact that the laws of physics are time translation invariant, not if you're talking about billions of years or a trillionth of a trillionth of a trillionth of a second after the big bang during the era of cosmic inflation

> The trouble with Sean's glib response to the question is that in each branch of the multiverse, we can measure the energy both before and after the supposed split.

Neither before or after the split are you measuring the absolute total energy in anything, in any energy measurement you're measuring the relative energy of something against a standard measure. If you say a particle has X units of energy calibrated against some standard measure, then after a measurement (and thus after a split) if you want to measure the energy in the decay products of the particle you do it by comparing them against the same standard measure, but that's impossible because any act of measurement splits a universe. So both the energy in the decay products and the energy of the standard measure of energy are decreased by 1/2 (or by however many times the universe splits), so you still get  X units of Energy and the world still looks the same to you despite it having only half the total absolute energy. It all comes down to the fact that you never measure the absolute energy of something, you always measure the relative energy.

> If I want to check energy conservation in neutron decay, I compare the mass-energy of the original neutron to the sum of the mass-energies of the decay products plus any kinetic energy of these decay products.

You left out a few steps. You compare the mass-energy of the original neutron against a standard calibration measure, and then you measure the mass-energies of the decay products plus any kinetic energy of these decay products against a standard measure. If the energy of the decay products and the energy of the standard measure have both been decreased by 1/2 then you're going to get the same units of energy both before and after the measurement.  

John K Clark    See what's on my new list at  Extropolis
7ff

John Clark

unread,
Apr 25, 2022, 7:09:26 AM4/25/22
to 'Brent Meeker' via Everything List


On Mon, Apr 25, 2022 at 1:48 AM Alan Grayson <agrays...@gmail.com> wrote:

> While I was at JPL[...]

How odd, JPL is in the habit of hiring people who don't know what any bright 10 year old knows, that you need hypersonic speeds to get into earth orbit, and yet JPL still somehow managed to perform the intricate gravitational calculations necessary to send a probe to Pluto. And I'm not impressed that you managed to find a list of Carl Sagan's papers, anyone with even minimal Internet skills can find them in about 20 seconds, but in all of those papers nobody can find the words "Alan Grayson'' anywhere in them.


John K Clark    See what's on my new list at  Extropolis
gas



Bruce Kellett

unread,
Apr 25, 2022, 8:08:30 AM4/25/22
to Everything List
On Mon, Apr 25, 2022 at 7:31 PM John Clark <johnk...@gmail.com> wrote:
On Sun, Apr 24, 2022 at 9:09 PM Bruce Kellett <bhkel...@gmail.com> wrote:

> The trouble with Sean's glib response to the question is that in each branch of the multiverse, we can measure the energy both before and after the supposed split.

Neither before or after the split are you measuring the absolute total energy in anything, in any energy measurement you're measuring the relative energy of something against a standard measure. If you say a particle has X units of energy calibrated against some standard measure, then after a measurement (and thus after a split) if you want to measure the energy in the decay products of the particle you do it by comparing them against the same standard measure, but that's impossible because any act of measurement splits a universe. So both the energy in the decay products and the energy of the standard measure of energy are decreased by 1/2 (or by however many times the universe splits), so you still get  X units of Energy and the world still looks the same to you despite it having only half the total absolute energy. It all comes down to the fact that you never measure the absolute energy of something, you always measure the relative energy.

You appear to be assuming that one measures energy against some reference energy. So that if both your reference and the thing you are measuring change by the same factor, you do not see any difference. That is true enough, but we do not always measure energy by comparison with some reference energy. Sometimes we use other laws of physics. For example, most of the energy in our immediate environment is mass energy, coming from the relation E = mc^2. So we can consider mass as a surrogate for energy. Mass can routinely be measured by weighing, assuming that the gravitational constant does not change. So if all energies halve, say due to a spin measurement, and we weigh our object before and after the split, in order to get the same result on the scales, the force due to gravity has to double in order for half the mass to give the same reading as before. But the force due to gravity depends on the local acceleration, g, and that depends on the mass of the earth, which also halves in this scenario. So, rather than the force of gravity doubling, it also halves, and the reading on our scales after the split is only 1/4 what it was before. If you think that such a change in the mass energy around us would not be noticeable, then you are not looking closely enough.


> If I want to check energy conservation in neutron decay, I compare the mass-energy of the original neutron to the sum of the mass-energies of the decay products plus any kinetic energy of these decay products.

You left out a few steps. You compare the mass-energy of the original neutron against a standard calibration measure, and then you measure the mass-energies of the decay products plus any kinetic energy of these decay products against a standard measure.

But that standard measure may not simply be another energy or mass. It could be the force on a charge in an electric field, or the measure on a spring balance in the gravitational field. If the local change in energy is to go unnoticed, then all the laws of physics must change in concert. It does not seem that the Schrodinger equation itself is able to accomplish this.

Bruce

Alan Grayson

unread,
Apr 25, 2022, 9:31:54 AM4/25/22
to Everything List
You can't seem to read and understand simple English. "Alan Grayson" is my pseudo-name of choice, and I could prove I was co-author on those papers, but why should I indulge an abusive schmuck? AG 

John Clark

unread,
Apr 25, 2022, 12:02:14 PM4/25/22
to 'Brent Meeker' via Everything List
On Mon, Apr 25, 2022 at 8:08 AM Bruce Kellett <bhkel...@gmail.com> wrote:

> You appear to be assuming that one measures energy against some reference energy. So that if both your reference and the thing you are measuring change by the same factor, you do not see any difference.

Yes, there must always be some sort of an energy standard if you're going to make any sort of energy measurement. That's why utility workers can safely grab a half million volt power line with both hands provided they make sure they're not grounded and are at the same voltage potential as the powerline, their right and left hands are at the same voltage potential so no current flows between them through their bodies, so the worker feels nothing and is perfectly safe, but if he were to touch a ground wire he'd be instantly fried. The same thing would happen if somebody in a high energy universe touched a wire that led to a lower energy universe, but fortunately for the inhabitants of both universes there is no way for that to happen.  

> That is true enough, but we do not always measure energy by comparison with some reference energy. Sometimes we use other laws of physics. For example, most of the energy in our immediate environment is mass energy, coming from the relation E = mc^2. So we can consider mass as a surrogate for energy. Mass can routinely be measured by weighing, assuming that the gravitational constant does not change.

The only reason we think the gravitational constant does not change is because when we measure the potential gravitational energy in something today against a standard calibration energy we find that we get the same number of energy units that we got yesterday when we measured the potential gravitational energy it was in against a standard calibration energy. But if the gravitational potential energy dropped by 90% and the calibration energy also dropped by 90% then we'd notice no difference and get the same number of units of energy both yesterday and today.  

> But that standard measure may not simply be another energy or mass. It could be the force on a charge in an electric field,

Electrical potential energy and gravitational potential energy are both energy, and if you want to measure either one you're going to need an energy calibration standard to do it. And the same is true for nuclear potential energy.  
 
> or the measure on a spring balance in the gravitational field.

It doesn't matter what you use, you're going to need an energy calibration standard because there's just no way to measure the absolute energy of anything, you can only measure the relative energy.  

John K Clark    See what's on my new list at  Extropolis
87u



John Clark

unread,
Apr 25, 2022, 12:05:49 PM4/25/22
to 'Brent Meeker' via Everything List
On Mon, Apr 25, 2022 at 9:31 AM Alan Grayson <agrays...@gmail.com> wrote:

> "Alan Grayson" is my pseudo-name of choice,

Sure it is.  

>  and I could prove I was co-author on those papers, but [...]

Sure you can.  
John K Clark    See what's on my new list at  Extropolis
ga0




Alan Grayson

unread,
Apr 25, 2022, 12:46:05 PM4/25/22
to Everything List
On Monday, April 25, 2022 at 10:05:49 AM UTC-6 johnk...@gmail.com wrote:
On Mon, Apr 25, 2022 at 9:31 AM Alan Grayson <agrays...@gmail.com> wrote:

> "Alan Grayson" is my pseudo-name of choice,

Sure it is.  

>  and I could prove I was co-author on those papers, but [...]
Sure you can.  

Any publications? Any degrees? You must have done SOMETHING!  Time to come clean. AG 

John Clark

unread,
Apr 25, 2022, 12:55:56 PM4/25/22
to 'Brent Meeker' via Everything List
On Mon, Apr 25, 2022 at 12:46 PM Alan Grayson <agrays...@gmail.com> wrote:

> Any publications? Any degrees? You must have done SOMETHING!  Time to come clean. AG 

Nothing special. Unlike you I never cleaned to work at JPL and I was never best friends with Carl Sagan, I'm just a humble electrical engineer and bookworm. But at least I know you need to obtain hypersonic speed to get into orbit, and flying saucer men in Roswell New Mexico do not excite me.  

John K Clark    See what's on my new list at  Extropolis
csb


Alan Grayson

unread,
Apr 25, 2022, 1:20:25 PM4/25/22
to Everything List
Nothing humble about you. Sagan was my boss for 18 months. I never claimed he was a friend. What university did you attend? I am a Cornell graduate, School of Arts and Sciences. I had a physics course with Philip Morrison. During the Cuban Missile Crisis I correctly predicted the outcome, but still got a "gentleman's" C in my course on International Relations. That's Cornell (at its worst). AG

csb


spudb...@aol.com

unread,
Apr 25, 2022, 4:36:48 PM4/25/22
to bhkel...@gmail.com, everyth...@googlegroups.com
So, with the knowledge of entanglement, and nobody seems sure yet how it really works, how sure should we be in a multiverse, where each cosm is Causally Disconnected from every other? 

Could be an energy transference, could be, most likely an information highway in the sky (Delta Dawn), could be whatever, magic! My claim is we cannot know enough to decide because our equipment used to measure phenomena is too puny. Whether neutrino catching, or proton slamming, or freezing electrons, or yeah, telescopes. 

A universe that is just starting out however, via eternal inflation, versus this one, would have higher energy levels and be much smaller and thus, expanding outwards like my gut. Disconnection there, as something random or a safety feature? Are these all connected, no matter what the age and state? Hmm. Perhaps all that is beyond the Hubble Volume, banging about, some 13.7 to 42 5o 80 billion lightyear's away? Or, maybe not. 


--
You received this message because you are subscribed to the Google Groups "Everything List" group.
To unsubscribe from this group and stop receiving emails from it, send an email to everything-li...@googlegroups.com.
To view this discussion on the web visit
https://groups.google.com/d/msgid/everything-list/CAFxXSLS0umDD0xAr00bywm0HE_6Ukk-y74ACz7%3D86Q_ZygEWpg%40mail.gmail.com
.

spudb...@aol.com

unread,
Apr 25, 2022, 5:19:07 PM4/25/22
to agrays...@gmail.com, everyth...@googlegroups.com
Hmm. You AG obviously haven't seen the state of education today with its corrosive, racial, and sexually preferenced faith movements? The mentioning of the Missiles of October moment and predicting the outcome is an interesting claim. Me, being a wimp, except when its time to retaliate, would have demurred from JFK's placing of Jupiter missiles in Turkey. A provocation, yet what do I call Khrushchev's walling up of Berlin in 1961? Monday morning quarterbacking is what I do, and today is a monday. For Putin, as JC has objected to, I'd offer Putin the right to place hypersonic missiles in "Cuber," ah, Cuba like in 1962. Why? Face saving for Putin with the Russian establishment and population, in exchange for Ukraine withdrawal, and a Neutrality agreement, with whomever? 

My idea is, how many times can ya kill us, if once suffices? This doesn't mean we turn the other cheek, and yes since Putin + Xi want an arms race, we should oblige them. 

Russian,  Sarmat (ICBM) and and Khinzal (cruise hypersonic) weapons, also a pride booster.Yeah, Cub-er!


On the Udder Hand said da Cow...


I also read a lot from Brian Wang's website Next Big Future for both bio, med, energy, and weapons, foreign policy goings on. He keeps and eye on things. But this too, is a reco from an aggressive schmuck as myself, who looks at things in a nuanced and not necessarily, ideological. Balanced, no, nuanced, yes.  




Nothing humble about you. Sagan was my boss for 18 months. I never claimed he was a friend. What university did you attend? I am a Cornell graduate, School of Arts and Sciences. I had a physics course with Philip Morrison. During the Cuban Missile Crisis I correctly predicted the outcome, but still got a "gentleman's" C in my course on International Relations. That's Cornell (at its worst). AG



-----Original Message-----
From: Alan Grayson <agrays...@gmail.com>
To: Everything List <everyth...@googlegroups.com>
Sent: Mon, Apr 25, 2022 1:20 pm
Subject: Re: aiming to complete Everett's derivation of the Born Rule

--
You received this message because you are subscribed to the Google Groups "Everything List" group.
To unsubscribe from this group and stop receiving emails from it, send an email to everything-li...@googlegroups.com.
To view this discussion on the web visit

Bruce Kellett

unread,
Apr 25, 2022, 6:42:36 PM4/25/22
to Everything List
On Tue, Apr 26, 2022 at 2:02 AM John Clark <johnk...@gmail.com> wrote:
On Mon, Apr 25, 2022 at 8:08 AM Bruce Kellett <bhkel...@gmail.com> wrote:

> That is true enough, but we do not always measure energy by comparison with some reference energy. Sometimes we use other laws of physics. For example, most of the energy in our immediate environment is mass energy, coming from the relation E = mc^2. So we can consider mass as a surrogate for energy. Mass can routinely be measured by weighing, assuming that the gravitational constant does not change.

The only reason we think the gravitational constant does not change is because when we measure the potential gravitational energy in something today against a standard calibration energy we find that we get the same number of energy units that we got yesterday when we measured the potential gravitational energy it was in against a standard calibration energy.

Sure, a spring balance needs to be calibrated against some standard mass. But we do not calibrate every day. Once the scale is set, we assume that the spring constant or whatever remains the same, so that recalibration is not necessary. So if all energies (including mass) drop by 90%, we will be able to detect this as long as the spring constant does not also change by this amount. Springs tend to rely on the electromagnetic properties of metals, and these will not change just because we measure a spin component in the next room.

To take another simple example, I used a spring balance to compare a mass against the gravitational field, where I assumed that Newton's constant does not change on a spin measurement. If all energies (and masses) drop by 50% in each branch of the spin measurement, then the mass of the earth decreases by 50%, and the local acceleration due to gravity, g, also drops by 50%. Now consider a simple pendulum: the period of swing is T = 2*pi*sqr(L/g), where L is the length of the pendulum. If g drops by 50%, the period of the pendulum increases by a factor of sqrt(2). This increase can easily be measured against a clock that does not rely on local gravity, such as a spring clock, or a crystal clock.

So the idea that a change in the energy of a branch is not noticeable is false -- one can always devise an experiment that does not rely on comparison with standard weights before and after the split. Other physics comes into play, and there is no suggestion that Newton's constant, for example, is influenced by our spin measurement, so the increase in the period of the pendulum is certainly measurable, as is the change in weight of our bag of flour.

Bruce

George Kahrimanis

unread,
Apr 25, 2022, 6:55:55 PM4/25/22
to Everything List
On Monday, April 25, 2022 at 4:09:23 AM UTC+3 Bruce wrote:
Despite Carroll's protestations (and the similar protestations of others), energy cannot be conserved in the multiverse -- each split must duplicate the energy of the whole as many times as there are branches.

Thanks for the citation. From the discussion so far, there seems to be no meaning in adding energy from different universes, so this is neither right not wrong (therefore, nuts). I think that this idea becomes meaningful only if we consider something like the gravitational field of an electron in a double-slit interference experiment: Caroll's idea implies that field (outside the box) would be as if generated by the electron-as wave, without decoherence. I suggest that we look at the consequences of this conclusion, to assess the plausibility of the idea.

George K.

Alan Grayson

unread,
Apr 25, 2022, 7:26:59 PM4/25/22
to Everything List
On Monday, April 25, 2022 at 3:19:07 PM UTC-6 spudb...@aol.com wrote:
Hmm. You AG obviously haven't seen the state of education today with its corrosive, racial, and sexually preferenced faith movements? The mentioning of the Missiles of October moment and predicting the outcome is an interesting claim. Me, being a wimp, except when its time to retaliate, would have demurred from JFK's placing of Jupiter missiles in Turkey. A provocation, yet what do I call Khrushchev's walling up of Berlin in 1961? Monday morning quarterbacking is what I do, and today is a monday. For Putin, as JC has objected to, I'd offer Putin the right to place hypersonic missiles in "Cuber," ah, Cuba like in 1962. Why? Face saving for Putin with the Russian establishment and population, in exchange for Ukraine withdrawal, and a Neutrality agreement, with whomever? 

This won't work in the present situation. Firstly, Putin will never willingly withdraw. And it was already tried! Remember the Budapest Memoranum of 1994? The Russians can't be trusted, and any agreement allowing any territorial concessions would be Munich 2.0. In this situation the Russians will regroup and try another day to conquer Ukraine. The only answer is to continue the fight until the Russians realize that tanks, towed and self-propelled artillery are obsolete against Javalins, INLAWS, etc. and counter-battery artillery. Eventually, the Russian soldiers will become so demoralized that they'll refuse to fight. Putin will likely use chemical weapons and blame the Ukrainians for their use. But it won't be decisive, and IMO tactical nukes will probably not be used. AG

spudb...@aol.com

unread,
Apr 25, 2022, 8:17:22 PM4/25/22
to everyth...@googlegroups.com
Ok, thanks for your response, AG. You are forecasting that the EU and or the US and Canada will continue to send, antitank and anti-aircraft missiles into Ukraine. Are you concerned that this will be successful, even with a Russian onslaught against the insertion of these into the Ukraine? 
Do you see a Russian attempt to wreck NATO countries via a conventional roll forward as with the Soviets and the Fulda Gap push? The notion that the demoralized Russian soldiers just stopping the madness on their own, goes against what the Soviets did in WW2. The NKVD has their own barrage battalions of troops to ensure that no Russian soldier retreated from the front line. So why not FSB or GRU troops today? 

Now using Sarin or Tabun or Soman or VX against the Ukrainians would seemingly be countered by the Ukrainian troops fighting in donated tox suits, and the strategy would then bring the fighting as quick as possible to the Russian troops. This, guaranteeing a mix of those fighting protected versus unprotected. I am suspecting that outside of Spetznaz, most Russian draftees will no be so protected? 

I will still hold open the possibility of giving Vlad an out, but you could be totally accurate regarding the Russkie mindset. I am not sure what circumstances would produce an ouster of Vlad, if such is even conceivable? 


Alan Grayson

unread,
Apr 25, 2022, 8:38:05 PM4/25/22
to Everything List
On Monday, April 25, 2022 at 6:17:22 PM UTC-6 spudb...@aol.com wrote:
Ok, thanks for your response, AG. You are forecasting that the EU and or the US and Canada will continue to send, antitank and anti-aircraft missiles into Ukraine. Are you concerned that this will be successful, even with a Russian onslaught against the insertion of these into the Ukraine? 
Do you see a Russian attempt to wreck NATO countries via a conventional roll forward as with the Soviets and the Fulda Gap push? The notion that the demoralized Russian soldiers just stopping the madness on their own, goes against what the Soviets did in WW2. The NKVD has their own barrage battalions of troops to ensure that no Russian soldier retreated from the front line. So why not FSB or GRU troops today? 

Now using Sarin or Tabun or Soman or VX against the Ukrainians would seemingly be countered by the Ukrainian troops fighting in donated tox suits, and the strategy would then bring the fighting as quick as possible to the Russian troops. This, guaranteeing a mix of those fighting protected versus unprotected. I am suspecting that outside of Spetznaz, most Russian draftees will no be so protected? 

I will still hold open the possibility of giving Vlad an out, but you could be totally accurate regarding the Russkie mindset. I am not sure what circumstances would produce an ouster of Vlad, if such is even conceivable? 


-----Original Message-----
From: Alan Grayson <agrays...@gmail.com>
To: Everything List <everyth...@googlegroups.com>
Sent: Mon, Apr 25, 2022 7:26 pm
Subject: Re: aiming to complete Everett's derivation of the Born Rule



On Monday, April 25, 2022 at 3:19:07 PM UTC-6 spudb...@aol.com wrote:
Hmm. You AG obviously haven't seen the state of education today with its corrosive, racial, and sexually preferenced faith movements? The mentioning of the Missiles of October moment and predicting the outcome is an interesting claim. Me, being a wimp, except when its time to retaliate, would have demurred from JFK's placing of Jupiter missiles in Turkey. A provocation, yet what do I call Khrushchev's walling up of Berlin in 1961? Monday morning quarterbacking is what I do, and today is a monday. For Putin, as JC has objected to, I'd offer Putin the right to place hypersonic missiles in "Cuber," ah, Cuba like in 1962. Why? Face saving for Putin with the Russian establishment and population, in exchange for Ukraine withdrawal, and a Neutrality agreement, with whomever? 

This won't work in the present situation. Firstly, Putin will never willingly withdraw. And it was already tried! Remember the Budapest Memoranum of 1994? The Russians can't be trusted, and any agreement allowing any territorial concessions would be Munich 2.0. In this situation the Russians will regroup and try another day to conquer Ukraine. The only answer is to continue the fight until the Russians realize that tanks, towed and self-propelled artillery are obsolete against Javalins, INLAWS, etc. and counter-battery artillery. Eventually, the Russian soldiers will become so demoralized that they'll refuse to fight. Putin will likely use chemical weapons and blame the Ukrainians for their use. But it won't be decisive, and IMO tactical nukes will probably not be used. AG

I don't think the experience of WW2 is particularly relevant to the current situation where Russia is the invader. AG

John Clark

unread,
Apr 25, 2022, 9:07:24 PM4/25/22
to 'Brent Meeker' via Everything List
On Mon, Apr 25, 2022 at 6:42 PM Bruce Kellett <bhkel...@gmail.com> wrote:

>> The only reason we think the gravitational constant does not change is because when we measure the potential gravitational energy in something today against a standard calibration energy we find that we get the same number of energy units that we got yesterday when we measured the potential gravitational energy it was in against a standard calibration energy.

> Sure, a spring balance needs to be calibrated against some standard mass. But we do not calibrate every day. Once the scale is set, we assume that the spring constant or whatever remains the same, so that recalibration is not necessary.

You're right, it's not necessary because as long as the test mass and the mass standard decrease by an equal percentage you're always gonna get the same result and you'll never notice that anything has changed. 

> So if all energies (including mass) drop by 90%, we will be able to detect this as long as the spring constant does not also change by this amount. Springs tend to rely on the electromagnetic properties of metals, and these will not change just because we measure a spin component in the next room.

If Many Worlds is correct then of course the spring constant will change because the world will split due to ANY measurement, and the absolute non-relative amount of energy of EVERY type will decrease. 

> I used a spring balance to compare a mass against the gravitational field, where I assumed that Newton's constant does not change on a spin measurement. If all energies (and masses) drop by 50% in each branch of the spin measurement, then the mass of the earth decreases by 50%, and the local acceleration due to gravity, g, also drops by 50%. Now consider a simple pendulum: the period of swing is T = 2*pi*sqr(L/g), where L is the length of the pendulum. If g drops by 50%,[...]

But g does NOT drop by 50% and I never said it did, I said the gravitational potential energy drops by 50%, and that will happen if the mass/energy of a gravitationally bound system drops by 50% even if g remains constant. If yesterday I measured the mass/energy of a pendulum and of the entire earth against an energy standard and I measure those things again today against today's energy standard, and if the mass/energy of the pendulum and the earth and today's energy standard have all decreased by 50%, then I will get the same measured value that I got yesterday even if g really is the same as it was yesterday.

And yes the force that the earth is pulling down on that pendulum would only be half as strong as it was yesterday, HOWEVER  the inertia (which is proportional to the mass/energy) of the pendulum would only be half as much as it was yesterday, so the two changes with cancel out and the pendulum would fall with the same acceleration that it did yesterday, and the period of its swing would be the same too.    


John K Clark    See what's on my new list at  Extropolis

maq

spudb...@aol.com

unread,
Apr 25, 2022, 9:18:43 PM4/25/22
to agrays...@gmail.com, everyth...@googlegroups.com
NLAW which was based on the AT-4 which is Swedish, is great for urban fighting as you indicate, but get it out overland and ...?


Brent Meeker

unread,
Apr 25, 2022, 9:33:17 PM4/25/22
to everyth...@googlegroups.com


On 4/25/2022 6:06 PM, John Clark wrote:
On Mon, Apr 25, 2022 at 6:42 PM Bruce Kellett <bhkel...@gmail.com> wrote:

>> The only reason we think the gravitational constant does not change is because when we measure the potential gravitational energy in something today against a standard calibration energy we find that we get the same number of energy units that we got yesterday when we measured the potential gravitational energy it was in against a standard calibration energy.

> Sure, a spring balance needs to be calibrated against some standard mass. But we do not calibrate every day. Once the scale is set, we assume that the spring constant or whatever remains the same, so that recalibration is not necessary.

You're right, it's not necessary because as long as the test mass and the mass standard decrease by an equal percentage you're always gonna get the same result and you'll never notice that anything has changed. 

> So if all energies (including mass) drop by 90%, we will be able to detect this as long as the spring constant does not also change by this amount. Springs tend to rely on the electromagnetic properties of metals, and these will not change just because we measure a spin component in the next room.

If Many Worlds is correct then of course the spring constant will change because the world will split due to ANY measurement, and the absolute non-relative amount of energy of EVERY type will decrease. 

> I used a spring balance to compare a mass against the gravitational field, where I assumed that Newton's constant does not change on a spin measurement. If all energies (and masses) drop by 50% in each branch of the spin measurement, then the mass of the earth decreases by 50%, and the local acceleration due to gravity, g, also drops by 50%. Now consider a simple pendulum: the period of swing is T = 2*pi*sqr(L/g), where L is the length of the pendulum. If g drops by 50%,[...]

But g does NOT drop by 50% and I never said it did, I said the gravitational potential energy drops by 50%, and that will happen if the mass/energy of a gravitationally bound system drops by 50% even if g remains constant. If yesterday I measured the mass/energy of a pendulum and of the entire earth against an energy standard and I measure those things again today against today's energy standard, and if the mass/energy of the pendulum and the earth and today's energy standard have all decreased by 50%, then I will get the same measured value that I got yesterday even if g really is the same as it was yesterday.

If all mass were scaled down by the same factor the gravitational interactions, like orbits and pendulums, would seem unchanged.  But what about the natural frequency of spring-mass systems?  Halving the mass while the EM forces between molecules of the spring stay the same means the frequency will go up.   So must all interaction constants change to save the appearance?

Brent


And yes the force that the earth is pulling down on that pendulum would only be half as strong as it was yesterday, HOWEVER  the inertia (which is proportional to the mass/energy) of the pendulum would only be half as much as it was yesterday, so the two changes with cancel out and the pendulum would fall with the same acceleration that it did yesterday, and the period of its swing would be the same too.    

John K Clark    See what's on my new list at  Extropolis

maq
--
You received this message because you are subscribed to the Google Groups "Everything List" group.
To unsubscribe from this group and stop receiving emails from it, send an email to everything-li...@googlegroups.com.
It is loading more messages.
0 new messages